btstudy.com 으로 오세요. 수능/내신 변형, 퀴즈를 무료로 공개합니다.

블루티쳐학원 | 등록번호: 762-94-00693 | 중고등 영어 | 수강료: 30(중등), 33(고등), 3+4(특강)

THE BLUET

728x90
반응형

20ss | Since 2005 임희재 | 블루티쳐학원 | 01033383436 | 200710 22:53:52

 

순서배열

 

1. 20수특 1-1

 

My name is Susan and I work in the accounting department.


(A) It is obvious to both me and the rest of our colleagues that she has a natural talent for what she does. Her clients love her and so do we. She is always quick to help with whatever we need no matter what it is. 1

(B) I happened to hear that you are looking to promote someone within our company to the position of manager of the accounting department, and I felt a need to write you this letter. I think Victoria Kimball, my department colleague, is an ideal candidate for the position. She is always on time, she never leaves early, and she does excellent work while she is here. 0

(C) She is a fantastic employee and deserves the position. You would not regret choosing her. As you are thinking about who might best fit into the position, I hope you will consider Victoria. 2


① A-C-B ② B-A-C ③ B-C-A ④ C-A-B ⑤ C-B-A

 


2. 20수특 1-2

 

You took a month-long leave of absence starting from November to December.


(A) As it was a month-long leave, it was obviously a part of your duties to hand over your work to your team members. Hence, it was very disappointing to discover that you went on the leave without giving them any information about the projects that you were handling. 0

(B) As you know, any work loss because of a failure to hand over work is a serious problem. This letter is the final warning regarding your irresponsible behavior. Failure to fulfill your duties in the future will not be tolerated and will result in us taking more severe actions. 2

(C) In turn, your carelessness led to unfortunate delays in the projects. We are sure that you are aware that during one's absence from work, the company cannot afford to put any work on hold. 1


① A-C-B ② B-A-C ③ B-C-A ④ C-A-B ⑤ C-B-A

 


3. 20수특 1-3

 

My name is David Lee and I'm a third-year chemistry major at Arizona State University.


(A) My résumé is attached in case you are interested. I look forward to hearing from you. 2

(B) While completing both my introductory and upper-level coursework, I've developed a passion for science and am extremely interested in pursuing independent research as an undergraduate. Personally, I am especially interested in nanotechnology. Recently I read your 2017 papers on the potential applications of graphene and became fascinated by your work. 0

(C) In particular, I found it amazing that graphene can be utilized to make flexible, transparent solar cells that can turn virtually any surface into a source of electrical power. If possible, I would be honored to work on a long-term project in your lab. Would you be interested in including me as part of your research team? 1


① A-C-B ② B-A-C ③ B-C-A ④ C-A-B ⑤ C-B-A

 


4. 20수특 1-4

 

We would like to express our deepest gratitude for considering AGL as your insurance provider.


(A) We hope that you will still consider taking out a policy with us when it becomes available. We appreciate your patience and understanding on this matter. 2

(B) Currently we are working on bringing that policy back. Therefore, we are keeping your application on hold for future reference and further processing when your preferred policy becomes available again. If you would prefer not to wait for approval and would like us to remove your application from our system, please let us know. 1

(C) You have made a wise choice, and your business is very important to us. Unfortunately, however, we are afraid to inform you that your insurance policy application cannot be approved at the moment. The type of policy you applied for isn't available right now. 0


① A-C-B ② B-A-C ③ B-C-A ④ C-A-B ⑤ C-B-A

 


5. 20수특 2-1

 

I was once out in the foothills of the Absaroka Mountains near my home in southern Montana when I saw a front of windy and snowy weather coming toward me.


(A) All I could do for some period of time was to crouch down and wait. 2

(B) Because the bare hillside was steep and already snow covered, the going was slow, so I didn't make it off the slippery slope before the blizzard hit. The wind hit with such force that I couldn't stand upright, and there was so much falling and blowing snow that I couldn't see much either. 1

(C) Because of the open nature of that part of the upper Yellowstone River Valley, I was able to see the storm coming from a long distance away, but I was so far out on an exposed hillside that I wasn't able to make it to cover before the blizzard hit. The slope where I was hiking was vegetated only with grass and sagebrush, so I started for a north-facing and timbered slope a mile or so away. 0


① A-C-B ② B-A-C ③ B-C-A ④ C-A-B ⑤ C-B-A

 


6. 20수특 2-2

 

My first morning in New York, I put on my girl-writer dress and heels and went to meet my editor, looking forward to a promising day.


(A) I kept touching my forehead, the way you pat your head to make sure your hair is okay. Then I started to cry and told him I had to go right that very second. He told me to phone him the next day. I said I would, although I had no intention of actually doing so. 2

(B) I looked at him quizzically. "I am so, so sorry," he said. "But it still doesn't work." I sat there staring at him as if his face were melting. 1

(C) I figured we would start editing together that very morning, and then he could give me the last of the advance. It would turn out that I had bounced back from this devastating setback and that truth and beauty had once again triumphed. Everyone would be so shocked to hear that this book had almost been thrown away. But my hopes were shattered when my editor said, "I'm sorry." 0


① A-C-B ② B-A-C ③ B-C-A ④ C-A-B ⑤ C-B-A

 


7. 20수특 2-3

 

In those more hopeful and innocent days there was much to enjoy about being newly a queen – not only that I had survived against the odds to inherit.


(A) The grand garment smelt faintly of beeswax and the softest leather. Unable to resist, I buried my features in it and laughed with delight. 1

(B) It was thrilling to know that such luxury was mine. My ladies laughed with me, equally delighted. They could not wear the elegant clothes and jewelry that as queen I wore even a queen with her hair as yet unbrushed, but their circumstances as my attendants were vastly better now that I was queen. 2

(C) I still remember the sensation of the royal white fur collar around my neck. I put my cheek to it and the fur seemed strangely warm against my skin. 0


① A-C-B ② B-A-C ③ B-C-A ④ C-A-B ⑤ C-B-A

 


8. 20수특 2-4

 

About three months ago, my sister was having problems with her daughter, Amy, so my sister and I decided to have Amy move in with my family.


(A) Instead of remaining supportive of her when she would get excited, I minimized her feelings by telling her to grow up. I expected her to understand the cause of her fears. Also, I expected her to magically allow herself to be a part of a loving family. It wasn't that simple for her, and I should not have reacted to Amy's emotions. 1

(B) We thought she could benefit from a chance of environment. Sadly, almost instantly, my relationship with Amy started to fall apart. She tried to verbalize her need for limits and boundaries in the home and for time spent alone together. However, our conversations quickly spiraled out of control. 0

(C) She needed a safe space to calm down, but I didn't give her that and didn't listen to her better. I should have respected her perspective and demonstrated that I was understanding of her fears. I should have allowed her to adjust to sharing space with my family. 2


① A-C-B ② B-A-C ③ B-C-A ④ C-A-B ⑤ C-B-A

 


9. 20수특 3-1

 

You are much more than just a list of your accomplishments.


(A) Or will you push yourself to perform like an all-star, the legendary player you know you can be, and dive for the ball? Your special effort will be recorded in the minds and memories of the others. More importantly, you will have the peace of mind and self-respect that come from knowing that you gave your very best – even if you don't end up making the catch. That's the stuff of legends. 2

(B) It says you give 110%. Every performance in your life becomes a choice you have to make. Will you do just enough to get by, letting the ball drop in front of you for a base hit? 1

(C) It's not only what you do, but how you do it that counts in the real scorebook of your life. Making the spectacular diving catch says more about you than the "out" that is recorded in the scorebook. It says you have game. 0


① A-C-B ② B-A-C ③ B-C-A ④ C-A-B ⑤ C-B-A

 


10. 20수특 3-2

 

Irrational acts don't just sabotage us.


(A) The television mini-series Lonesome Dove was a big hit in the 1980s. Two rugged Texas Rangers shared a life together that eventually led them from Texas to Montana, where they made their fortune from a cattle drive. One partner died and the other promised to personally return the body to Texas; an incredible sacrifice. 1

(B) From rational point of view, the dead partner wouldn't know where he was buried; he was already dead. The surviving partner's friends thought his promise was foolish. They pressured him to ship the boy to Texas by train. No rational argument would ever win this battle, and if it had, the movie would have lost its charm. 2

(C) They can also make us heroes, lovers, and generous helpers. The qualities we admire most in others are their emotional ones, not their intellectual ones. Rarely does intellect alone inspire romantic acts or heroic deeds. 0


① A-C-B ② B-A-C ③ B-C-A ④ C-A-B ⑤ C-B-A

 


11. 20수특 3-3

 

Ideas or theories about human nature have a unique place in the sciences.


(A) Forty years ago, the distinguished anthropologist Clifford Geertz said that human beings are "unfinished animals." What he meant is that it is human nature to have a human nature that is very much the product of the society that surrounds us. That human nature is more created than discovered. 1

(B) We "design" human nature, by designing the institutions within which people live. So we must ask ourselves just what kind of a human nature we want to help design. 2

(C) We don't have to worry that the cosmos will be changed by our theories about the cosmos. The planets really don't care what we think or how we theorize about them. But we do have to worry that human nature will be changed by our theories of human nature. 0


① A-C-B ② B-A-C ③ B-C-A ④ C-A-B ⑤ C-B-A

 


12. 20수특 3-4

 

Near my old office building, the window of a show store advertised the generous offer of a free shoe shine.


(A) I had to even the score, somehow. Since I didn't need shoes, I found myself mindlessly looking at shoe trees, laces, and polish. Finally, I quietly walked out of the store empty-handed and uneasy. Even though I had managed to escape from the store, I was sure many others were not so fortunate. 2

(B) Free was free, he said. I climbed down from the chair feeling distinctly indebted. "How could this guy shine my shoes," I thought, "and expect nothing?" So I did what I suspect most people who take the offer do – I looked around for something to buy. 1

(C) I walked by this tore dozens of times and thought nothing of it. One day, though, with my shoes looking a little scuffed and some time on my hands, I decided to avail myself of this small bounty. After my shine, I offered the shoeshine man a tip. He refused. 0


① A-C-B ② B-A-C ③ B-C-A ④ C-A-B ⑤ C-B-A

 


13. 20수특 4-1

 

People everywhere have their special cultural rites.


(A) Physicians scrub for seven minutes before doing a surgical procedure. While the necessity of the prolonged scrub is open to question with the advent of modern germicides, its traditional role in preparing the surgical team for a delicate procedure is undeniable. In the airline business, the first officer deplanes the aircraft and conducts a walk-around inspection before takeoff. 1

(B) It is just as true in the workplace as it is in the surrounding culture. All professions observe their distinctive ways. What appears to be a superficial set of actions to outsiders knits insiders together and puts them in an appropriate frame of mind to do their work successfully. 0

(C) Very seldom do they discover something wrong. But symbolically it prepares the cockpit crew for their awesome responsibility of getting all the souls aboard safely to their destinations. 2


① A-C-B ② B-A-C ③ B-C-A ④ C-A-B ⑤ C-B-A

 


14. 20수특 4-2

 

From what I have seen in counseling, the besetting sin of the father as performance-focused parent is his misguided belief that dogged disapproval will cause his teenagers to try harder and do better.


(A) And the father contaminates his connection with his teenager by provoking his or her increased resentment and dislike. "Dad is never satisfied, no matter how I do, which is all he really cares about!" As a performance coach, a father would be far better served by praising the good than by only faulting mistakes in the misguided belief that his expressions of dissatisfaction will cause improvement. 2

(B) The paternal criticism is offensive, not appreciated. The teen wants less to do with him and for him, not more. 1

(C) "I'll keep criticizing you until your attitude and motivation improve!" In fact, excessive criticism only hurts the teenager's feelings and discourages him or her from wanting to hear what the father has to say and from doing what the father wants him or her to do. 0


① A-C-B ② B-A-C ③ B-C-A ④ C-A-B ⑤ C-B-A

 


15. 20수특 4-3

 

When a young police officer puts on a uniform for the first time, it almost certainly feels strange and foreign.


(A) Equally, they are powerful statements to ourselves about what to expect of ourselves. This, together with the way other people react to our appearance, powerfully shapes how we feel, think and behave. 2

(B) they help people think themselves into a particular way of behaving, and communicate clearly to other people what function that person is expected to perform. Our dress and appearance are a sort of uniform as well, whether we like it or not. They are very powerful statements to other people about what to expect from us. 1

(C) Yet other people react to that uniform in a range of more or less predictable ways ― just as they do to a priest or to a white-coated doctor. These reactions help to make the police officer feel a part of the uniform and more comfortable with the role that goes with it. This is the point of uniforms:. 0


① A-C-B ② B-A-C ③ B-C-A ④ C-A-B ⑤ C-B-A

 


16. 20수특 4-4

 

While individualism gives strength, it also can create a weakness if not moderated by involvement with others ― family, friends, and society.


(A) Pure individualism may lead to a philosophy of convenience and a lack of participation in or appreciation of the civic and social process. This limits personal growth, mutes gaining leadership skills and traits, and deprives one of the true contexts of life that is the reality model of one's mind. 0

(B) We look at the world as centered upon us and give ourselves undue influence on the reality of events. We need to move into a position of not thinking the world rotates around us but how we fit into the world. This is the concept of context. 2

(C) The biggest problem the world now has is that we do not interact personally much anymore. Our technology removes some of reality and replaces it with perception, relativity, and inherent self-interest often pushing us to convenience. 1


① A-C-B ② B-A-C ③ B-C-A ④ C-A-B ⑤ C-B-A

 


17. 20수특 4-5

 

By taking a conscious interest in your brain and how it works, and by consciously trying some of the techniques that follow, you can acquire a variety of useful mental skills.


(A) A cross section of our society represents the "average" level of thinking skill ― the level one might expect of a large number of people who have never thought very much about thinking. To move beyond the level of average thinking, you must think about thinking. 1

(B) You must pay attention to how your brain works, and you must experiment with new techniques. Once you do that, you will clearly see the value of it. 2

(C) It isn't really very difficult, but it won't happen by accident. The prevalence of negative thinking, fuzzy and illogical thinking, and rigid thinking in our society proves that fact that these higher-level thinking skills do not come naturally. 0


① A-C-B ② B-A-C ③ B-C-A ④ C-A-B ⑤ C-B-A

 


18. 20수특 4-6

 

The term statistical significance is an unfortunate choice of words.


(A) But too many people hear the phrase "statistically significant" and assume it also suggests that the results are significantly important. That may or may not be true. 1

(B) Instead, think of the term as suggesting that you would expect to find the same results 95 out of 100 times if a study is replicated in a similar manner or 90 out of 100 times, depending on what measure of reliability is used. True significance lies in interpreting the data correctly to ensure that it has meaning or importance for the organization you represent. 2

(C) But it's part of our research vocabulary, and it will continue to appear in reports. It refers to the fact that the results discovered, or differences between two sets of data, could reliably be expected to occur again if another study was conducted in a similar manner. 0


① A-C-B ② B-A-C ③ B-C-A ④ C-A-B ⑤ C-B-A

 


19. 20수특 4-7

 

The survival of wilderness ― of places that we do not change, where we allow the existence even of creatures we perceive as dangerous ― is necessary.


(A) And I would argue that we do not need just the great public wildernesses, but millions of small private or semiprivate ones. Every farm should have one; wildernesses can occupy corners of factory grounds and city lots ― places where nature is given a free hand, where no human work is done, where people go only as guests. 1

(B) Our sanity probably requires it. Whether we go to those places or not, we need to know that they exist. 0

(C) These places function, I think, whether we intend them to or not, as sacred groves ― places we respect and leave alone, not because we understand well what goes on there, but because we do not. 2


① A-C-B ② B-A-C ③ B-C-A ④ C-A-B ⑤ C-B-A

 


20. 20수특 4-8

 

Facing your difficulty head-on is the first positive step in the process of fixing it.


(A) Follow the remedies they prescribe. If some project you are working hard to finish on time encounters severe problems, examine the difficulty as a scientist would. What caused the problem? What are the options? Try to discover the best ways of dealing with the realities you face, focus on what will be most beneficial, then act accordingly. 2

(B) Your promotion went to someone else ― now what? Face it directly. There must be a reason why your boss selected the other person. Get your hurt and anger under control and go find out why you didn't get the promotion. 0

(C) Ask your boss what you need to do to improve yourself so you'll be considered more seriously next time. If you have just learned that you have a health problem, face it squarely and intelligently. What is the best treatment? Ask the top specialists for their advice. 1


① A-C-B ② B-A-C ③ B-C-A ④ C-A-B ⑤ C-B-A

 


21. 20수특 5-1

 

Finding an ideal location for a piano is often difficult.


(A) In the order of importance, the location should help preserve the instrument, be acoustically satisfactory, and be aesthetically pleasing. Ideally, a piano should be placed on an inside wall, away from the direct rays of the sun. 0

(B) Instruments that are placed directly beneath water pipes or emergency sprinkler systems should be protected with a water proof cover from possible water damage. Finding the best location for a piano also includes acoustical considerations; usually a piano sounds best in a room without thick wall-to-wall carpeting or heavy, sound-absorbing draperies. 2

(C) Moreover, it should not be placed next to heaters, stoves, air conditioners, or near heat ducts or cold air returns. Drafty locations next to open windows or doors should also be avoided. 1


① A-C-B ② B-A-C ③ B-C-A ④ C-A-B ⑤ C-B-A

 


22. 20수특 5-2

 

Power, considered by some theorists to be the "entrance requirement" for anger, is not necessary for sadness.


(A) Anger is an "approach" emotion, while sadness is a "retreat" emotion. Thinking of a person as sad makes us see them as weaker and more submissive. Anger, not sadness, is associated with controlling one's circumstances, such as competition, independence, and leadership. 0

(B) Like happy people, angry people are more optimistic, feeling that change is possible and that they can influence outcomes. Sad and fearful people tend toward pessimism, feeling powerless to make change. 2

(C) Anger, not sadness, is linked to assertiveness, persistence, and aggressiveness. Anger, not sadness, is a way to actively make change and confront challenges. Anger, not sadness, leads to perceptions of higher status and respect. 1


① A-C-B ② B-A-C ③ B-C-A ④ C-A-B ⑤ C-B-A

 


23. 20수특 5-3

 

Sadly enough, some of us have distorted lessons of happiness that developed in our childhood.


(A) Most believe attaining true happiness is like winning the lottery, and only some of us are lucky enough to win it. Or maybe some of us believe in "works of righteousness" ― a theology that says if you work hard enough at anything, you will receive what you work for. 1

(B) Any one of these theories of happiness is born of the philosophy that happiness is scarce. Looking around our world right now I would have to agree that true happiness is in short supply. But this is because we have bought into a belief system that teaches us that happiness is as scarce as hen's teeth. 2

(C) Our experiences developed as we grew up in different systems, such as our original family, our religious community, and our neighborhood. Many of us believe that only a few of us experience true happiness. 0


① A-C-B ② B-A-C ③ B-C-A ④ C-A-B ⑤ C-B-A

 


24. 20수특 5-4

 

The causes and consequences of war may have more to do with pathology than with politics, more to do with irrational pressures of pride and pain than with rational calculations of advantage and profit.


(A) it also suggests that if there is a root cause of human conflict and of the power drive of nations, it lies not in hopes of economic development, historical forces, or the workings of the balance of power, but in the ordinary hopes and fears of the human mind. 2

(B) What this "proves," if anything, is that computers are more rational than men;. 1

(C) There is a Washington story, perhaps apocryphal, that the military intellectuals in the Pentagon conducted an experiment in which they fed data derived from the events of the summer of 1914 into a computer and that, after weighing and digesting the evidence, the machine assured its users that there was no danger of war. 0


① A-C-B ② B-A-C ③ B-C-A ④ C-A-B ⑤ C-B-A

 


25. 20수특 6-1

 

Aging is an economic challenge because unless retirement ages are drastically increased so that older member of society can continue to contribute to the workforce (an economic imperative that has many economic benefits), the working-age population falls at the same time as the percentage of dependent elders increases.


(A) In addition, fewer people are likely to take entrepreneurial risks because aging workers tend to preserve the assets they need to retire comfortably rather than set up new businesses. 1

(B) As the population ages and there are fewer young adults, purchases of big-ticket items such as homes, furniture, cars and appliances decrease. 0

(C) This is somewhat balanced by people retiring and drawing down their accumulated savings, which in total lowers savings and investment rates. 2


① A-C-B ② B-A-C ③ B-C-A ④ C-A-B ⑤ C-B-A

 


26. 20수특 6-2

 

During the 1890s Richard Henry was caretaker of Resolution Island in remote Fiordland on the west coast of New Zealand's South Island.


(A) A keen naturalist, he noted with concern the impact on native birds of the arrival of recently introduced stoats as they invaded this last corner of New Zealand. In a desperate attempt to protect populations of the flightless kakapo and little spotted kiwi between 1894 and 1900 he translocated hundreds of individuals from the mainland on to Resolution Island. 0

(B) Nevertheless, the technique of isolating species in danger on predator-free islands that may or may not have been occupied by the species in the past became a vital tool to prevent extinctions from predation by introduced predators in New Zealand. 2

(C) Unfortunately, Resolution was too close to the mainland and stoats invaded in 1900. Thus, Henry's efforts were in vain. 1


① A-C-B ② B-A-C ③ B-C-A ④ C-A-B ⑤ C-B-A

 


27. 20수특 6-3

 

Genes give us the foundation of our models.


(A) It is our job as supportive adults to find a constructive purpose. This does not mean that we should view violent behavior as resourceful; rather, we can enlist the core of violent behavior as a positive resource (eg, Violent behavior may exemplify an eagerness to take control, an ability to respond authoritatively, or a refusal to be victimized). As yourself in what context or situation the core of a particular behavior would signify value. 1

(B) Experiences give us individual identities. Behaviors express our individual needs, desires, urges, attitudes, beliefs, and so on. In this way, all behaviors are purposeful. 0

(C) For example, "Your refusal to be victimized will help you grow more tolerant with people as you mature." This comment orients the child toward a more fulfilling future because it validates the child's world view and enlists the core of the behavior as a positive resource. 2


① A-C-B ② B-A-C ③ B-C-A ④ C-A-B ⑤ C-B-A

 


28. 20수특 6-4

 

Managing relationships is a value that represents how someone considers the interests and well-being of other people involved in a person's social world.


(A) Being a host, guest or co-worker also shapes food choice situations where roles and relationships are primary considerations in food choice. 2

(B) When people provide food for others, share food with others or receive food from others, they typically consider the needs, preferences and feelings of those people related to what, how, when and where food is eaten. Personal needs and preferences are often compromised to build, maintain or repair relationships. 0

(C) Food is central to family harmony, and someone who adopts the role of the 'household food manger' is typically very attentive to the preferences, dislikes and patterns of eating of others. For example, newly married couples must negotiate ways to make joint food choices and parent-child relationships contribute to constructing family food decisions. 1


① A-C-B ② B-A-C ③ B-C-A ④ C-A-B ⑤ C-B-A

 


29. 20수특 8-1

 

Kate Seredy was born in 1899, in Budapest, Hungary.


(A) Seredy illustrated two children's books in Hungary before moving to the United States in 1922. She supported herself in her new homeland by illustrating lamp shades and greeting cards. As her knowledge of English increase, she found work illustrating textbooks and children's trade books. 1

(B) Her father, a teacher, helped her develop an appreciation for books. After high school she earned an art teacher's diploma from the Academy of Arts in Budapest and also spent time studying in Italy, France, and Germany. She served as a nurse for two years during World War I, and the pacifist stance she subsequently developed later influenced her writing. 0

(C) Seredy wrote her first book, The Good Master, after an editor suggested she try writing about her childhood in Hungary. Seredy won the Newbery Medal in 1938 for The White Stag, a book based on legends about the founding of Hungary that her father told her as a child. 2


① A-C-B ② B-A-C ③ B-C-A ④ C-A-B ⑤ C-B-A

 


30. 20수특 8-2

 

Eastern cottontails are the most common rabbits in North America.


(A) A pregnant female digs a shallow hole, which is deeper at one end than the other. She lines the nest with grass and fur from her belly. 2

(B) The name "cottontail" is derived from their short, rounded tails, which have white fur on their underside. Easter cottontail rabbits do not dig burrows, although they may shelter in disused ones dug by other animals. Unlike hares, which rely on their speed to outrun predators, cottontails freeze when under threat, blending into their surroundings. 0

(C) If they have to run, they follow zigzag paths, attempting to shake off their pursuers. In warmer parts of their range cottontails breed all year round, but farther north breeding is restricted to summer. Males fight to establish hierarchies, with top males getting their choice of mates. 1


① A-C-B ② B-A-C ③ B-C-A ④ C-A-B ⑤ C-B-A

 


31. 20수특 8-6

 

Born on February 9, 1874, Amy Lowell was the last of the five children of Augustus and Katherine Bigelow Lawrence Lowell, who resided in a mansion on a ten-acre estate in Brookline, Massachusetts.


(A) Although Lowell dabbled in verse from an early age, it was a chance encounter with Leigh Hunt's Imagination and Fancy in her father's collection that inspired her serious interest in poetry. During this same interval she discovered the poetry of John Keats, whose work would profoundly influence her aesthetics. 1

(B) Lowell's lifelong appreciation of Keats's writings, especially their correlation of beauty and human longing, was summed up in her two-volume study of the poet, published in the last year of her life. 2

(C) Privately schooled in Boston until the age of seventeen, Lowell learned little of literature during her formal education. She fostered her literary interests by reading in the extensive library at home and at the Boston Athenaeum. 0


① A-C-B ② B-A-C ③ B-C-A ④ C-A-B ⑤ C-B-A

 


32. 20수특 8-7

 

On March 20, 1883, Jan Matzeliger (1852-1889) patented the first successful shoe-lasting machine.


(A) The next year he settled in nearby Lynn, Massachusetts. There he developed his device while working in a shoe factory. The machine increased productivity as much as fourteen times over hand methods and led to concentration in the industry. 1

(B) Matzeliger continued to work on the machine to improve its quality, and received a patent for a third and improved model on March 20, 1883. He invented a number of other devices, including a mechanism for distributing tacks and nails. 2

(C) Matzeliger was born in Surinam of a Dutch father who was an engineer and a black mother who was Surinamese and probably came from West Africa. He left Surinam in 1871 and became a sailor on an East Indian ship. Matzeliger settled in Philadelphia for a while, holding odd jobs until he moved to Boston in 1876. 0


① A-C-B ② B-A-C ③ B-C-A ④ C-A-B ⑤ C-B-A

 


33. 20수특 9-1

 

Thomas Edison failed ten thousand times until, like a bolt of lightening, the solution to the challenge of the incandescent light bulb hit him, and because of this, he transformed the world.


(A) Rosa Parks had no idea that choosing to resist her conditioning ― the injustice of racial prejudicing ― and risking imprisonment, beating, or even worse, would spark a revolution that would transform history for all men and women, regardless of race. When Mother Teresa chose to leave her position educating the privileged classes of India in order to care for the poor and forgotten, people thought that she was crazy. 0

(B) So, too, will you receive your aha moments of brilliance, unexpectedly and through no effort of your own. 2

(C) At the time, Mother Teresa had no idea of the global impact that this decision would have on the world. Albert Einstein received the solution to his Theory of Relativity while dreaming that he was riding on a beam of light. 1


① A-C-B ② B-A-C ③ B-C-A ④ C-A-B ⑤ C-B-A

 


34. 20수특 9-2

 

The major themes of country music are work, freedom, and alienation, appearing in approximately one-sixth of all Top 20 selections.


(A) Consequently, from the start country music was an eclectic mix, which included ancient British ballads, Americanized versions of these, sacred songs, minstrel tunes, early blues, and songs of many sorts absorbed from the commercial popular music industry over the years. 1

(B) Over the 20th century it has grown from a homegrown and heartfelt music, expressing working-class identity, into a commercial music, produced by others to appeal to a working-class identity, whether or not its listeners are actually working class. The symbolic meaning of country music as the declared favorite music of New England-educated, upper-class President George Bush ― presenting him as a "regular American" ― is unmistakable. 2

(C) Country music is itself a commercial offshoot of the traditional song and instrumental music of the South, formerly carried on in an oral tradition. 0


① A-C-B ② B-A-C ③ B-C-A ④ C-A-B ⑤ C-B-A

 


35. 20수특 9-3

 

We must understand that busyness does not necessarily equal productivity, and in order to accomplish anything in life, we must prioritize.


(A) What we put at the top of our list determines how we spend our time and how much value we get out of each and every day. The truth is, most of us are unconscious about how we truly spend our time, prioritizing things that we say don't really matter to us. 0

(B) We do so much in a day, yet wonder where all of our time went. Once we get honest about how we're actually spending our days, we can begin to prioritize what's most important to us, and start taking back our time. 2

(C) We waste our extra time watching TV, complaining, or sleeping our lives away. We mindlessly scroll through social media and spend hours talking on the phone, and we don't make good use of our time traveling to and from work. 1


① A-C-B ② B-A-C ③ B-C-A ④ C-A-B ⑤ C-B-A

 


36. 20수특 9-4

 

The term genius can be traced back to the Latin word ingenium: a natural-born talent.


(A) It was not until the Renaissance that people began to describe an artistic creative potential or the source of inspiration as genius. The key significance for invention is that the so-called genius develops ideas that no one has had previously and, in the words of Immanuel Kant, that 'genius must be considered the very opposite of a spirit of imitation'. 1

(B) The essence of this talent is seen as original productivity, which employs confident intuition to access new areas of creativity. The person who has genius ― a brilliant creative power ― is also known as a genius. 0

(C) In addition, Kant established that genius 'cannot indicate scientifically how it brings about its product, but rather gives the rule as nature. Hence, where an author owes a product to his genius, he does not himself know how he conceived the ideas, nor is it in his power to invent the like at pleasure, or methodically, and communicate the same to others in such precepts as would put them in a position to produce similar products. 2


① A-C-B ② B-A-C ③ B-C-A ④ C-A-B ⑤ C-B-A

 


37. 20수특 9-5

 

The way our society views weight and getting into shape is one example of where expectations are often out of touch with reality, and lead to impatience ― one of the top reasons why over 95 percent of all diets fail.


(A) We search for the latest fads and are drawn to products claiming to trim our waistline in thirty days or less. Yet time after time we find ourselves, after attempts to diet, with ever more weight to lose. 1

(B) While there are many factors that go into weight loss and healthy living, one of the hardest things for people to accept is that any real lifestyle change takes time: time to break bad habits and time to form new healthy ones. 2

(C) We are led to believe through various media that dropping pounds is something that can be done quickly and effectively if we try hard enough. The reality show The Biggest Loser, where contestants drop ten or even twenty pounds in one week, implies that rapid results are not only possible but commonplace. 0


① A-C-B ② B-A-C ③ B-C-A ④ C-A-B ⑤ C-B-A

 


38. 20수특 9-6

 

Recent psychological research has revealed that college students who look at a two-second video clip of a professor teaching can predict how students who spend an entire semester with that professor will like that professor by the end of the semester.


(A) That statement is then recorded. At the end of the semester, students who have taken a class with the professor anonymously record whether or not they liked the professor. 1

(B) In other words, a student watching a two-second clip of a professor says, "I like him." Or "I don't like him." 0

(C) With incredible accuracy, those watching the two-second clip predict what the entire class will feel at the end of the semester. It sounds unbelievable, but it is true. One of the big mistakes we make is making a poor impression on others. 2


① A-C-B ② B-A-C ③ B-C-A ④ C-A-B ⑤ C-B-A

 


39. 20수특 9-7

 

Imagination and creativity are the gate keys of fantasy role-playing.


(A) The students exercise their imagination and creativity in countless ways, from taking on the role of their assigned characters to interacting with other creatures and alien environments. 1

(B) If students cannot imagine themselves engaged by the fantasy world described to them, then the game cannot get off the ground. 0

(C) In every case, what is minimally called for is imaginative flexibility in order to react appropriately to the multiple situations the students encounter, while looking ahead to the consequences of various actions and decisions. This means that fantasy role-playing provides an ideal environment to cultivate and test the productive use of imagination, utilizing it to enliven the fantasy narrative, envision alternatives, and empathize with others. 2


① A-C-B ② B-A-C ③ B-C-A ④ C-A-B ⑤ C-B-A

 


40. 20수특 9-8

 

Advertising is a form of persuasion.


(A) I don't have to be the first to break it to you that this is not the main concern of an advertising copywriter for a home appliance company. He's unlikely to suffer a single pang of failure upon finding out that, by buying the washer in his beautifully-crafted ad, you've passed up a wiser purchase. 1

(B) If you really want to read something that's written with your best interests in mind, you pick up a copy of Consumer Reports ― you don't go flipping through Good Housekeeping to find that dishwasher ad you saw last week. When you do read the ad, you take it for granted that any comparisons it makes to the competitions are not necessarily "fair and balanced." 2

(C) This means that from square one, your goals and interests are often very different from those of the advertiser. Let's say your goal is to buy the best dishwashing machine you can afford. 0


① A-C-B ② B-A-C ③ B-C-A ④ C-A-B ⑤ C-B-A

 


41. 20수특 10-1

 

Yale psychologist Irving Janis showed that just about every group develops an agreed-upon view of things — a consensus reality, the "PC" or politically correct view.


(A) In his classic book, Groupthink, Janis explained how panels of experts made enormous mistakes. People on the panels, he said, worry about their personal relevance and effectiveness, and feel that if they deviate too far from the consensus, they will not be taken seriously. 1

(B) Any evidence to the contrary is automatically rejected without consideration, often ridiculed, and may lead to exclusion of the person presenting the un-PC data. So group members are careful not to rock the boat by disagreeing with the consensus — doing so can seriously damage their standing. 0

(C) People compete for stature, and the ideas often just tag along. Groupthink causes groups to get locked into their course of action, unable to explore alternatives, because no one questions the established course. The more cohesive the group, the greater the urge of the group members to avoid creating any discord. 2


① A-C-B ② B-A-C ③ B-C-A ④ C-A-B ⑤ C-B-A

 


42. 20수특 10-2

 

The temperature of 54°F appears to be a magic threshold for several species.


(A) Because to really make a decent sound, the air temperature must be at least 54°F. If it is cooler, you'll barely hear a squeak from these tiny musicians.1

(B) For example, at the height of summer, billowing meadows full of grasses and herbs are the habitat for grasshoppers and crickets, which provide an orchestral backdrop with their chirping. However, this soundscape is by no means constant. 0

(C) As cold-blooded creatures, grasshoppers can't regulate their own body temperature, and only really get going when it's warm enough. Their body movements become faster with rising temperatures, resulting in ever more rapid vibrations of the legs and wings, which produce the chirping sound, depending on the type. This also changes the frequency of the tone produced: the warmer it is, the higher the pitch. 2


① A-C-B ② B-A-C ③ B-C-A ④ C-A-B ⑤ C-B-A

 


43. 20수특 10-3

 

In America we have developed the Corporation Man.


(A) In the areas of management, sales, and public relations, the position of the corporation man is secure only from one stockholders' meeting to the next; a successful rebellion there may sweep out whole cadres of earnest men and replace them with others. 2

(B) His position in the pyramid of management is exactly defined by the size of his salary and bonuses. The pressures toward conformity are subtle but irresistible, for his position and his hopes for promotion are keyed to performance of duties, activities, and even attitudes which make the corporation successful. 1

(C) His life, his family, and his future lie with his corporation. His training, his social life, the kind of car he drives, the clothes he and his wife wear, the neighborhood he lives in, and the kind and cost of his house and furniture are all dictated by his corporate status. 0


① A-C-B ② B-A-C ③ B-C-A ④ C-A-B ⑤ C-B-A

 


44. 20수특 10-4

 

Unfortunately, there are some social scientists who refuse to admit the limitations of their field of study.


(A) The use of mathematical techniques is not an end in itself but only a means to an end, namely, the discovery of what's true about the material world). The use of numbers is one way to be more precise in our effort to rationally understand causes. 2

(B) We cannot really quantify prejudice or love, for instance. When all is said and done, such attempted quantification is in vain. What is often forgotten, even in the physical sciences, is that science is not primarily a matter of quantification. 1

(C) They push hard to make social science imitate physical science. This is usually done by the use of all sorts of numbers, tables, charts, and graphs in order to give the impression of a profound quantification of the subject matter. Now, as a matter of fact, some things can be quantified and some things cannot. 0


① A-C-B ② B-A-C ③ B-C-A ④ C-A-B ⑤ C-B-A

 


45. 20수특 10-5

 

Culture consists of the linked stock of ideas that define a set of commonsense beliefs about what is right, what is natural, what works.


(A) Today, many serious-minded citizens seek to make important issues out of the potential disappearance of the whooping crane, the timber wolf, and other exotic creatures." 2

(B) These commonsense beliefs are not universal, but are instead typically bounded by time as well as by space. Today's orthodoxy may be the heterodoxy of yesterday and tomorrow. 0

(C) Although cultural change is not usually perceptible from day to day, when we look over a longer time span it becomes apparent that even the most fundamental assumptions about morality and the standards by which quality of life should be evaluated are subject to change. In his 1972 paper, Anthony Downs offers a vivid illustration of the extent of cultural change with his observation that "One hundred years ago, white Americans were eliminating whole Indian tribes without a blink. 1


① A-C-B ② B-A-C ③ B-C-A ④ C-A-B ⑤ C-B-A

 


46. 20수특 10-6

 

There is the question of the innate musicality of humans.


(A) Not all can attain professional proficiency;. but then, although all humans can learn to speak, not all can become great orators. 1

(B) We know that all normal humans inherit the ability to learn language — it is somehow "hardwired" in the human brain — but whether the same is true of music, whether all humans are basically musical is not clear, in part because cultures differ so much in their conception of "singing." Yet it seems likely that all humans can learn to sing minimally, to beat rhythms accurately, and to recognize simple pieces. 0

(C) The world's societies differ in the degree to which they encourage individuals to participate in music. In some rural societies, most people are considered about equally good at singing, and everyone participates in music-making at public events. In many urban societies, musical participation is largely limited to listening to live music and even more to recordings, whereas performance is left to professionals. 2


① A-C-B ② B-A-C ③ B-C-A ④ C-A-B ⑤ C-B-A

 


47. 20수특 10-7

 

Graham Allison intriguingly laid out his ideas about the Thucydides Trap, tensions between an established power and a rising one, in an essay for The Atlantic.


(A) As part of the Thucydides Trap Project, a team under Allison's direction examined sixteen cases where a rising power challenged an established one and determined the outcome of such challenges. The results are disturbing. Fourteen out of sixteen cases resulted in war. 0

(B) Both Taiwan and Japan have the potential to draw the US closer to war with China. Arguably, the US's allies might play a bigger role in the Thucydides Trap than fear of a rising power. This results in a situation where a great deal of effort is required to escape the Thucydides Trap. 2

(C) The worrying factor, according to Allison, is the fact that normal events or 'standard crises' that can otherwise be resolved, nevertheless trigger war. In the case of Athens and Sparta, it was the actions of smaller allies that drew them closer to war. This can easily happen in East Asia. 1


① A-C-B ② B-A-C ③ B-C-A ④ C-A-B ⑤ C-B-A

 


48. 20수특 10-8

 

Mummification in Ancient Egypt was developed in response to a gradual change in the burial preferences of its deceased.


(A) The physical features of the body would be retained, and this lifelike appearance of the corpse may have supported the belief of an afterlife. As burial practices became more sophisticated, with the construction of elaborate tombs and monuments to the dead, the bodies of the deceased were no longer buried in the desert sand. 1

(B) However, as belief in the afterlife and rebirth was fundamental to Egyptian burial practices, mummification was developed to artificially preserve the body in readiness for the journey to the underworld and to be judged by Osiris. 2

(C) The artificial preservation of bodies, both human and animal, was practised in Egypt from about 2686 BC until the beginning of the Christian era. The earliest Egyptians were buried in the sand, typically in the foetal position to reduce the size of the hole that would need to be dug, and the hot, dry climate would dehydrate the body. 0


① A-C-B ② B-A-C ③ B-C-A ④ C-A-B ⑤ C-B-A

 


49. 20수특 11-1

 

Fos Whitlock was a successful businessman.


(A) He asked me to talk to his son, Brant. Brant was about to graduate from college and was obsessed with the idea that he had the potential to be a professional golfer. Fos asked me to meet with Brant, obviously hoping that I would steer him toward giving up his dream of being a professional golfer. 0

(B) What was worse: The possibility of risking some money by supporting his son or running the risk of his son's lifelong enmity? To his great credit, Fos did an about-face and made what I think was the right choice. 2

(C) After seeing Brant, I met with Fos and suggested that he had too much to lose. If Brant were deprived of the chance to see if he had the right stuff to become a pro golfer, he would never know if he could have been successful. He would then hate his father for the rest of his life for depriving him of this opportunity. 1


① A-C-B ② B-A-C ③ B-C-A ④ C-A-B ⑤ C-B-A

 


50. 20수특 11-2

 

"Do you like the garden, Ms.


(A) Danby?" Charlotte spoke softly, her voice no more than a dry whisper, but Sarah had no difficulty hearing her. The accent was more noticeable than it had first been on the telephone. "Yes" she said with a thin smile. "I like it very much." 0

(B) Once inside, was instantly drawn to the artful blend of fine decoration in this private sector of being the house. It was just like the garden, filled with simplicity and colour, much like the former owner, she suspected. Each piece of furniture filled its rightful place with pride. The paintings on the walls, every one obviously a masterpiece, were each unique. She recognised a few celebrated names, which meant the collection had grown over the years. 2

(C) Charlotte smiled appreciatively. She pulled open the door and gestured with her hand. "Please, won't you come in?" The smile instantly vanished. Sarah allowed Charlotte to lead the way. 1


① A-C-B ② B-A-C ③ B-C-A ④ C-A-B ⑤ C-B-A

 


51. 20수특 11-3

 

There was Hungry Anderson, who was known to be a tight man with a dollar.


(A) He was called Hungry Anderson from that day on, and people began to say he was a miser. To prove that he wasn't, he bought a shiny Chalmers automobile, but his instincts were too strong for him. He kept the car in a shed in town and came in with his horse and buggy, motored about town, put up the car, and trotted back to his farm. 2

(B) He and his wife lived about a mile out of town. He got his name on an occasion when he had a carpenter working on the roof of his house. 0

(C) At noon, it took the carpenter about six or seven minutes to get down off the roof, and by the time he did, Hungry had eaten his lunch. He explained that when the carpenter was late, he had thought he didn't want to eat. 1


① A-C-B ② B-A-C ③ B-C-A ④ C-A-B ⑤ C-B-A

 


52. 20수특 11-4

 

The Prussian king, Wilhelm I, was a conventional man, and not particularly clever or insightful—.


(A) in other words, the opposite of his minister, Bismarck. He disliked much of what Bismarck did and how he did it. Yet at some level, the king recognized that he and his dynasty needed Bismarck even though, as Wilhelm once mildly complained, "It's hard to be Kaiser under him." 0

(B) He frequently threatened to resign. In the end it was always Wilhelm who backed down saying, "Bismarck mustn't think of resigning." Wilhelm wrote to him after one scene:. "It is my greatest happiness to live with you and thoroughly agree with you!" 2

(C) Since the monarch had the final say over foreign and defence policy, and governments answered only to him, and not the other way around, Bismarck only in the name of Wilhelm was able to exert great control over domestic and foreign affairs. The two men's relationship was marked by terrible arguments. Bismarck would come down with severe headaches and fits of vomiting and claim that he was dying. 1


① A-C-B ② B-A-C ③ B-C-A ④ C-A-B ⑤ C-B-A

 


53. 20수특 12-1

 

You may have noticed that people differ in the schemas they tend to use when evaluating others.


(A) As these examples illustrate, the role of the evaluator or the context in which a target person is encountered often influences which traits or schemas are used. 1

(B) College professors are often concerned with whether someone is smart, sales managers with whether someone is persuasive, and those involved in the entertainment business with whether someone has charisma. 0

(C) But sometimes the schema is simply determined by habit: if a person uses a particular schema frequently, it may become chronically accessible and therefore likely to be used still more frequently in the future. A frequently activated schema functions much like a recently activated one: its heightened accessibility increases the likelihood that it will be applied to understanding a new stimulus. 2


① A-C-B ② B-A-C ③ B-C-A ④ C-A-B ⑤ C-B-A

 


54. 20수특 12-2

 

Not everyone comes to see the game.


(A) This very point was elaborated by William McDougall in a book published in 1908, in which he developed his case for the existence of a gregarious instinct in humans. The question he posed at the turn of the century certainly argues convincingly for the view that we are social creatures. 1

(B) For some, the contest merely provides the setting and opportunities for the expression of other motives. The social contact provided by the crowd itself suggests a reason for people to attend. 0

(C) However, additionally labeling the observed behavior as an "instinct" creates a tautology that adds nothing to our understanding. McDougall asks his readers, "What proportion of the ten thousand witnesses of a football match would stand for an hour or more in the wind and rain, if each man were isolated from the rest of the crowd and saw only the players?" We would guess very few in 1908, fewer today. 2


① A-C-B ② B-A-C ③ B-C-A ④ C-A-B ⑤ C-B-A

 


55. 20수특 12-3

 

There is a widespread belief that creativity is best served through inner peace, stillness, and calmness.


(A) The way she put it to me was, "I have ninety minutes when Sam is napping, and I run to the computer and write like crazy. I'm totally focused." Turns out, my colleague is onto something. In fact, it is better to be aroused when attempting to think creatively. 2

(B) The result? She became prolifically productive. In her words, she was "wired." 1

(C) One of my colleagues was convinced that her own creative writing was best when she had no distractions, quietly sipping tea in a peaceful setting. However, after three months of such languid writing days, she produced nothing that she was proud of. Shortly thereafter, her first baby was born and her schedule went from long, open, peaceful, unstructured days to tightly orchestrated, minute-by-minute slots tightly orchestrated, minute-by-minute slots, punctuated by extreme activity. 0


① A-C-B ② B-A-C ③ B-C-A ④ C-A-B ⑤ C-B-A

 


56. 20수특 12-4

 

During a particularly trying time early in my sales career, a sales manager gave me a poster that read, "If it is to be, it is up to me."


(A) You cannot authoritatively impose effective change upon any society;. rather, it must be a choice that is felt deep within the consciousness of its inhabitants. 1

(B) I realized within that moment that if any changes or improvements were going to occur in my outside world, they needed to begin within my inside world. In life, all meaningful or macrocosmic change within society begins on a microcosmic level — that's you and me. 0

(C) Collectively, society's individuals must band together and say with one voice, "We're mad as hell and we're not gonna take it any more." For this to occur, however, one person must be willing to take the reins of change, with all of the risk those reins entail, and with an iron resolve, lead the charge of effective change. 2


① A-C-B ② B-A-C ③ B-C-A ④ C-A-B ⑤ C-B-A

 


57. 20수특 12-5

 

According to Greek mythology, the Oracle at Delphi was consulted to gauge the risk of waging a war.


(A) The opinions of everyone surveyed are summarized in a report and returned to the respondents, who then have the opportunity to modify their opinions. Because the written responses are kept anonymous, no one feels pressured to conform to anyone else's opinion. If people change their opinions, they must explain the reasons why; if they don't, they must also explain why. 1

(B) In modern times, the term Delphi refers to a group survey technique for combining the opinions of several people to develop a collective judgment. The technique comprises a series of structured questions and feedback reports. Each respondent is given a series of questions (eg, what are the five most significant risks in this project﹖), to which he writes his opinions and reasons. 0

(C) The process continues until the group reaches a collective opinion. Studies have proven the technique to be an effective way of reaching consensus. 2


① A-C-B ② B-A-C ③ B-C-A ④ C-A-B ⑤ C-B-A

 


58. 20수특 12-6

 

What is the basic idea of sociology?


(A) Then, too, neighbors of theirs — white children — had been their friends when they were four and five years old, but by the time they were fourteen and fifteen a barrier had gone up between them. My black undergraduates could see that this racial bias was hardly innate; rather, it showed that social structure affects how people think. Hence they were open to the sociological perspective. 2

(B) It is this: Social structure pushes people around, influences their careers, and even affects how they think. My Tougaloo College students readily understood that social structure pushed people around. 0

(C) Not one of their parents was an architect, for example, because no school in the Deep South in their parents' generation both taught architecture and admitted African Americans. So my Tougaloo students knew how social structure might influence careers. 1


① A-C-B ② B-A-C ③ B-C-A ④ C-A-B ⑤ C-B-A

 


59. 20수특 12-7

 

According to the scholars of the Indian traditions (Vedas), the origin of religion was to be sought in the impressions that natural phenomena made upon man.


(A) The primary stage of religion was not due to the religious nature of man, or to the 'need of the human heart', as O. Müller expressed it in his book, but to man's elementary capability of seeing personal figures in the impersonal phenomena of his surroundings. What finally led to the formation of religion was, thus, the elaboration of a nature mythology, and the veneration of the respective figures. 1

(B) The mythological figures were thought to be personifications of natural objects. The impressive manifestations of nature stimulated the personifying fantasy of man. 0

(C) The beginning of religion was the worship of many natural objects, with a predominance of such phenomena as the sun, the sky, thunderstorms, lightning, rain, and fire. 2


① A-C-B ② B-A-C ③ B-C-A ④ C-A-B ⑤ C-B-A

 


60. 20수특 12-8

 

A well-functioning democracy requires a media system that provides diverse sources of information and encourages civic participation.


(A) The first step is to break up the concentration of media power. Let's give control to a greater number of smaller companies that could legitimately compete with a broader range of information. 1

(B) The government once considered the airwaves such an integral part of our democracy that politicians decided the public should own and control them. It is time for the public to reclaim the responsibility of producing quality media from the corporate conglomerates. 0

(C) Also, we must create and maintain a noncommercial public media system as well as independent alternative media that exist outside the control of transnational corporations and advertisers. The rise of independent political blogs, and that of alternative podcasts, radio networks and television channels are all examples of citizens rising up to take back control of our media. 2


① A-C-B ② B-A-C ③ B-C-A ④ C-A-B ⑤ C-B-A

 


61. 20수특 12-9

 

Consider the question often asked by scientists, including even those who are well disposed toward animals, as to whether the hen suffers from what she has never known.


(A) Somewhat to her surprise, she found that hens who had been confined to battery cages, cages no larger than a sheet of newspaper, when given the choice between a small outside run with grass and the cages they had known all their lives, chose to stay in the cages. 1

(B) The Oxford researcher Marian Dawkins conducted experiments to determine what hens felt about their homes. 0

(C) And fowl expert Valerie Porter points out that chickens taken from a battery cage "will be in a considerable state of what you might call cultural shock if they are deprived of the only type of environment they have ever known. In fact, they will curl up in a corner in a state of terrified agoraphobia and it will take a great deal of time and patient understanding to rehabilitate them to real life." 2


① A-C-B ② B-A-C ③ B-C-A ④ C-A-B ⑤ C-B-A

 


62. 20수특 12-10

 

The cyclical nature of success and failure has been well established in the field of modern bridge design and engineering, in which experience spans about two centuries.


(A) Failures are part of the technological condition. 2

(B) The creative and inherently human process of design, upon which all technological development depends, is in effect timeless. What this means, in part, is that the same cognitive mistakes that were made three thousand, three hundred, or thirty years ago can be made again today, and can be expected to be made indefinitely into the future. 1

(C) Unfortunately, the lessons learned from failures are too often forgotten in the course of the renewed period of success that takes place in the context of technological advance. This masks the underlying fact that the design process now is fundamentally the same as the design process thirty, three hundred, even three thousand years ago. 0


① A-C-B ② B-A-C ③ B-C-A ④ C-A-B ⑤ C-B-A

 


63. 20수특 12-11

 

The popular press deems reshoring to be "bringing manufacturing back home..." from a current location that is not home.


(A) GE, for example, reshored its appliance manufacturing from its production facility in China to the US in its own plant to meet the US demand, whereas the US-based Vaniman Manufacturing decided to no longer buy sheet metal fabrication from an overseas supplier and to instead source from a local supplier to meet demand in the US. 1

(B) The term is agnostic as to whether the manufacturing being brought home occurred in a wholly owned facility in an offshore location or in the factory of an offshore supplier. 0

(C) Both would be considered reshoring more precisely, reshoring back to the US. Reshoring is fundamentally concerned with where manufacturing activities are to be performed, independent of who is performing the manufacturing activities in question — a location decision only, as opposed to a decision regarding location and ownership. 2


① A-C-B ② B-A-C ③ B-C-A ④ C-A-B ⑤ C-B-A

 


64. 20수특 12-12

 

Mark Leary and his colleagues led participants to believe that they were to perform a group task.


(A) Our momentary feelings of self-worth strongly depend on the extent to which others approve of us and include us. 2

(B) The experimenter ignored the participants' actual preferences and randomly assigned some participants to a condition in which they had supposedly been passed over by the others and had to work alone, and other participants to a condition in which they were in high demand by others and worked with a group. Participants in the work-alone condition, who believed they had been excluded, reported lower levels of self-esteem than those involved by the group. 1

(C) Before the task, each participant was asked to write an essay about "what it means to be me" and "the kind of person I would most like to be." The experimenter then gave each person's essay to other participants (in another location) who were asked to indicate who they would like to work with in the group setting. 0


① A-C-B ② B-A-C ③ B-C-A ④ C-A-B ⑤ C-B-A

 


65. 20수특 13-1

 

Movies and cartoons sometimes portray scientists as loners in white lab coats, working in isolated labs.


(A) In reality, science is an intensely social activity. Most scientists work in teams, which often include both graduate and undergraduate students. 0

(B) And, in fact, research papers aren't published until they are vetted by colleagues in what is called the "peer review" process. Most of the examples of scientific inquiry described in science textbooks for college students, for instance, have all been published in peer-reviewed journals. 2

(C) And to succeed in science, it helps to be a good communicator. Research results have no impact until shared with a community of peers through seminars, publications, and websites. 1


① A-C-B ② B-A-C ③ B-C-A ④ C-A-B ⑤ C-B-A

 


66. 20수특 13-2

 

The immune system is the body's defense against foreign invaders such as bacteria.


(A) We know that the immune system begins to decline after adolescence, and the weakening of immune function is linked to age-related vulnerability. 1

(B) According to the autoimmune theory of aging, the system may eventually become defective and no longer distinguish the body's own tissues from foreign tissues. The body may then begin to attack itself, as suggested by the rising incidence of autoimmune diseases with advancing age. 2

(C) The immune system protects and preserves the body's integrity, and it does this by developing antibodies to attack hostile invaders. 0


① A-C-B ② B-A-C ③ B-C-A ④ C-A-B ⑤ C-B-A

 


67. 20수특 13-3

 

When a painter is working for a patron, as Leonardo da Vinci did for the Medici, there can be emotional communication resulting from the artist's intention to produce something that the patron will appreciate and enjoy.


(A) So painting is a social process involving the communication of emotional judgments of the artist to the people who view it. Painters cannot expect viewers to appreciate their work with exactly the same perceptions and emotions that went into their creation, but they can hope to generate some approximation of these. 2

(B) Some artists often take pains concerning the framing and presentation of their work for the benefit of the viewers of their art, such as when Mark Rothko was highly particular about the lighting of his paintings. 1

(C) Even Vincent van Gogh, who was isolated and sold few paintings in his life, cared about the reactions of his brother to the paintings that he produced. 0


① A-C-B ② B-A-C ③ B-C-A ④ C-A-B ⑤ C-B-A

 


68. 20수특 13-4

 

Salt was historically so costly and important in Europe that its consumption was linked to social status.


(A) Among the privileges granted to the elite was access to salt, placed in a container on the high table. This clear expression of social divide is reflected in the phrases "above the salt," referring to someone of high rank, and "below the salt," which means someone of lower rank or less socially acceptable. 2

(B) In the medieval world, with its rigid hierarchy, the way in which people dined and the food that they ate reflected their position in society. 0

(C) Royalty and nobility sat at the high table, positioned on a dais, while their social inferiors ate at lower tables below them. 1


① A-C-B ② B-A-C ③ B-C-A ④ C-A-B ⑤ C-B-A

 


69. 20수특 13-5

 

If you stop to think about all of the tiny parts that make up any one thing, you could be thinking about it for quite some time.


(A) A computer, for example, looks like a big hunk of metal and glass, but if you look closer, moving your way down from the big things to the tiniest parts that make the computer up, you start to see how a computer is really many smaller things combined to make something bigger. 1

(B) A computer isn't just a computer: It's metal, glass and plastic, screws, springs, wires, copper, ink, electricity, and so much more. How those small parts come together to make the bigger parts is what matters, but how often do we take time to think about that fact? 2

(C) There are almost an infinite number of things that influence and impact any larger thing, right down to atoms and smaller molecules. 0


① A-C-B ② B-A-C ③ B-C-A ④ C-A-B ⑤ C-B-A

 


70. 20수특 13-6

 

Taub and his co-workers have already found strong evidence that the brain can be healed by its own plasticity.


(A) Similar results have been produced with speech impairment and even dyslexia. Although this research is still in its early days, Michael Merzenich of the University of California, San Francisco, believes the brain's plasticity may really enable us to protect ourselves against age decline. 2

(B) Amazingly, some people who have lost the use of an arm through a stroke have been trained to use it again by having the good arm restrained and being forced to use the apparently dead one, a technique called 'constraint-induced (CI) movement therapy'. 0

(C) Even though the part of the brain that controlled the arm was damaged, the CI movement therapy forced the brain to open up new areas in order to move the dead limb. 1


① A-C-B ② B-A-C ③ B-C-A ④ C-A-B ⑤ C-B-A

 


71. 20수특 13-7

 

When you watch a documentary on a small standard (4 x 3) video screen and then see it on an equally small wide-screen (16 x 9) receiver, you probably notice relatively little energy change.


(A) Even if you use proper conversion methods for aspect ratios, squeezing such large images into the small video screen reduces not only image size but also, if not especially, event energy. Close-ups, inductive sequencing, and a dense audio track help generate some aesthetic energy on the small video screen, but they cannot compete with the large movie images and high-volume surround sound. 2

(B) This is why some movies that emphasize landscape (from actual landscapes to spaceships or battle scenes) must be seen on the large screen to feel the total impact. 1

(C) When you switch from the small screen to a large HDTV screen, however, the energy change is readily apparent. 0


① A-C-B ② B-A-C ③ B-C-A ④ C-A-B ⑤ C-B-A

 


72. 20수특 13-8

 

Revegetation techniques are put in place with an understanding of the resultant effects on wildlife.


(A) Berry-producing plants should not be selected as they may prove attractive yet fatal to wildlife due to the close proximity of traffic. 2

(B) For instance, the planting of tree lines and woodland areas within roadside landscape may naturally force birds to fly higher above roads when crossing between forest edges, as was shown in the Netherlands. Verges should be widened and landscape planting on bends or curves on a road should be set back, to improve visibility and to discourage crossings by wildlife. 0

(C) In contrast, along straight sections of road carriageway, vegetative cover should extend as close to the road as permitted by road construction and safety standards. Landscaping of centre medians, junctions, roundabouts or interchanges should be sensitive to wildlife. 1


① A-C-B ② B-A-C ③ B-C-A ④ C-A-B ⑤ C-B-A

 


73. 20수특 14-1

 

A diverse garden will become a habitat for a variety of bird species.


(A) Or rather, not a time, but a certain position of the sun. These are relative to sunrise, a precisely definable event. 1

(B) Unfortunately, it changes constantly, as throughout spring, the sunrise takes place a little earlier each day, until the summer solstice on June 21, when it starts getting later again. So, bird song is perhaps not ideal as a genuine replacement for your watch, although each species tends to observe its relative time slot, day by day, with astonishing accuracy. 2

(C) But if all the birds were to sing at the same time, each one's melody would be drowned out in the cacophony of voices. In order for each singer to be adequately appreciated by his rivals or his sweetheart, each species focuses on a specific time in the morning. 0


① A-C-B ② B-A-C ③ B-C-A ④ C-A-B ⑤ C-B-A

 


74. 20수특 14-2

 

With reference to the variable of intensity, it is almost stating the obvious to say that bright lights or loud sounds can attract our attention.


(A) The experiment conducted by LaBarbera and MacLachlan exposed people to five radio commercials that were either normal or time-compressed on the order of 130%. These time-compressed commercials were not "sped up" by making the tape run faster; that would also increase the frequency of the auditory signal, and make the announcer sound like a high-pitched Mickey Mouse. 1

(B) Rather, the time-compression technique involves the shortening of pauses between words, and the reduction of the length of vowel sounds. This results in a message that runs more quickly, without changing the pitch of the announcer's voice. These researchers found that the time-compressed advertisements elicited more interest and better recall than the normal ads. 2

(C) We have all been exposed to countless examples of commercial advertisements that seem to be based solely upon this premise. One unusual example of the use of intensity in advertising contexts is the practice of time-compressed speech in radio commercials. 0


① A-C-B ② B-A-C ③ B-C-A ④ C-A-B ⑤ C-B-A

 


75. 20수특 14-3

 

Recycling means recovery and reprocessing of waste materials for use in new products.


(A) Waste radionuclides recovered from contaminated materials are difficult to recycle in new devices or compounds. Hence, even materials that contain large amounts of radioactive constituents (eg sealed radioactive sources as used in industry, medicine and research) often are immobilized (conditioned) and safely stored and disposed of rather than recycled. 2

(B) Due to their inherent radiation, radionuclides are much more difficult to recover from contaminated materials. Recovery usually presumes concentration of species into a smaller volume even though this may result in more dangerous materials. 1

(C) Recycled waste can be substituted for raw materials, reducing the quantities of wastes for disposal as well as potential pollution of air, water and land resulting from mineral extraction and waste disposal. However, recycling has certain limitations when applied to radioactive materials. 0


① A-C-B ② B-A-C ③ B-C-A ④ C-A-B ⑤ C-B-A

 


76. 20수특 14-4

 

Glass affords transparency.


(A) As a result, birds often try to fly through windows. And every year, numerous people injure themselves when they walk (or run) through closed glass doors or large picture windows. If an affordance or anti-affordance cannot be perceived, some means of signaling its presence is required. 2

(B) To be effective, affordances and anti-affordances have to be discoverable — perceivable. This poses a difficulty with glass. The reason we like glass is its relative invisibility, but this aspect, so useful in the normal window, also hides its anti-affordance property of blocking passage. 1

(C) At the same time, its physical structure blocks the passage of most physical objects. As a result, glass affords seeing through, but not the passage of air or most physical objects (atomic particles can pass through glass). The blockage of passage can be considered an anti-affordance — the prevention of interaction. 0


① A-C-B ② B-A-C ③ B-C-A ④ C-A-B ⑤ C-B-A

 


77. 20수특 14-5

 

In cultural contexts that require polite formulas rather than honest words, language may lose almost all its communicative function, and here food often takes over the role.


(A) More information about the actual social transactions going on at the dinner is transmitted by food choice and distribution. The most valued guest often gets the choicest portion, and so on down. 1

(B) Other aspects of the ritual may communicate even more. Everyone carefully observes who sits next to the host, who sits at the host's table, who is the first one to be greeted, who is served first, who gets the best piece of meat, or who is urged to have seconds. 2

(C) In formal dinners around the world, it is not usually appropriate to send the important social messages verbally. Words are bland and carefully chosen. 0


① A-C-B ② B-A-C ③ B-C-A ④ C-A-B ⑤ C-B-A

 


78. 20수특 14-6

 

Ritualistic behaviour designed to influence future events is not, it seems, limited to humans.


(A) Skinner's explanation for this phenomenon was that the accidental pairing of the release of food early on in the process with whatever the bird happened to be doing was enough to reinforce that particular type of activity. 2

(B) B. F. Skinner's classic research into 'superstition in the pigeon', conducted at Indiana University in 1948, supports this hypothesis. Skinner described an experiment in which pigeons were placed inside a box and were presented with a small piece of food once every fifteen seconds, regardless of their behaviour. 0

(C) After a few minutes the birds developed various little unusual rituals, such as walking round in circles, moving their heads up and down and so on. The pigeons appeared to have concluded that their little routines were causing the release of the food even though in reality there was no relationship whatsoever. 1


① A-C-B ② B-A-C ③ B-C-A ④ C-A-B ⑤ C-B-A

 


79. 20수특 14-7

 

Within the domain of concrete entities, objects and substances have very different properties.


(A) When we say that two objects are identical or the same, we are referring to two objects in their entirety and not to two distinctive parts of a single object. In contrast, when we say that two substances are identical or the same, there is no notion of wholeness. 1

(B) Substances are of scattered existence, and there is no such thing as whole sand, whole water, or whole clay. This portion of sand is identical to that portion of sand, as long as the two portions consist of the same physical constituents. This difference in identity or sameness between objects and substances leads to fundamentally different extension principles for determining category membership across the two ontological kinds. 2

(C) Objects are individuated, whereas substances are nonindividuated. Thus, the two kinds of entities have fundamentally different criteria for the notion of identity or sameness. 0


① A-C-B ② B-A-C ③ B-C-A ④ C-A-B ⑤ C-B-A

 


80. 20수특 14-8

 

Although the efforts to revive dying languages are admirable, the challenges facing those who would reverse the extinction process are intimidating.


(A) The recent revolution in communications technology has provided powerful tools (through the airwaves and cyberspace) for the spread of mainstream Western culture and language. Yet, for some endangered languages, the tide is changing through the digital revolution. 1

(B) Not all of the extinctions are the direct result of hostility and repression from a dominant government, as was the case with American Indians throughout most of U.S. history. But where brutal repression failed to make indigenous languages and culture extinct, intense globalization since the 1980s has been more successful. 0

(C) As Rosenberg points out, digital technology, discussion groups, software companies, and apps are lifelines for language preservation for minority and endangered language communication needs. At one time technology forced some language speakers to adopt the dominant language of their community or nation. Now, new tools create the possibility for revitalizing languages and retaining language speakers of endangered languages. 2


① A-C-B ② B-A-C ③ B-C-A ④ C-A-B ⑤ C-B-A

 


81. 20수특 15-1

 

Several studies have shown that individuals who are ostracized, excluded, or rejected by others behave in ways that will increase their chances of eventually becoming accepted.


(A) Similarly, Williams, Cheung, and Choi observed that ostracized individuals were more likely than others to conform to the opinions of other people. Thus, these studies show that in response to social rejection, people seek to reconnect themselves with their social worlds. 1

(B) In addition, Gardner, Pickett, and Brewer found that individuals who experience social rejection are more likely to remember socially relevant information. Thus, belongingness needs appear to guide the processing and retention of information that is consistent with one's motive. 2

(C) These behaviors range from working harder in group settings, to conforming to group perceptions, or being more sensitive to information about others. For example, Williams and Sommer found that women responded to ostracism by increasing their efforts on a subsequent group task. 0


① A-C-B ② B-A-C ③ B-C-A ④ C-A-B ⑤ C-B-A

 


82. 20수특 15-2

 

David Rock, author of Your Brain at Work, has described in fascinating detail the intricate mechanics of the brain on creativity and stress.


(A) In this condition, we feel curious, open-minded, and interested in what we are doing ― all excellent qualities for thriving on the job. Neuroscience tells us that creativity and engagement are essential to making people happier. 1

(B) We know, for example, that self-described happy people have more new ideas. We now know that stress decreases our cognitive resources, whereas mindfulness induces what is called a toward state in the brain, an openness to possibilities. 0

(C) But the technological onslaught of today's world can also become highly stressful. Long hours, hard work, and high pressure are made worse by our being permanently plugged in. Though the introduction of laptop computers, high-speed Internet, mobile technology, and social media have wonderful advantages in how we connect, they also reinforce behaviors that shut down the toward state and set us on autopilot. 2


① A-C-B ② B-A-C ③ B-C-A ④ C-A-B ⑤ C-B-A

 


83. 20수특 15-3

 

For sea squirts, a two-part life cycle provides a quite obvious advantage.


(A) But then, if the young sea squirts immediately settled down to the bottom, the sea squirt colony would soon be so crowded that they would have to grow on top of each other. There would not be enough food to feed the huge crowds of sea squirts, all jammed into a small area. So instead, the tadpole-like swimming larvae of the sea squirts do not settle down immediately. 1

(B) They swim and drift with the ocean currents. By the time they are ready to change to adults and take up a place on the ocean bottom, they have been scattered over a wide area. 2

(C) Adult sea squirts live very nicely, attached to the sea bottom. All the food they need comes drifting to them in the ocean currents, and they never have to move. They have even solved the problem of getting together to mate by shooting their sperms and eggs out into the water. 0


① A-C-B ② B-A-C ③ B-C-A ④ C-A-B ⑤ C-B-A

 


84. 20수특 15-4

 

The halo effect causes one trait about a person to color your attitude and perceptions of all her other traits.


(A) If the first year of a relationship is deeply fulfilling and life-altering, it can take a long time to notice if things turn sour later. If you like specific aspects of an individual, the halo effect causes the positive appraisal to spread to other measurements and to resist attack. 1

(B) Even stranger, the more noticeable the aspect is when you form your first impression, the more difficult it becomes to change your attitude about that aspect. So, for example, if you are bowled over by the warmth and kindness of a coworker in your first week at a new job, you'll let him get away with a host of obnoxious behaviors later on, maybe even for years. 0

(C) Beautiful people seem more intelligent, strong people seem nobler, friendly people seem more trustworthy, and so on. When they fall short, you forgive and defend them, sometimes unconsciously. 2


① A-C-B ② B-A-C ③ B-C-A ④ C-A-B ⑤ C-B-A

 


85. 20수특 15-5

 

Divers working at high pressures underwater usually breathe "air" that is a mixture of oxygen and helium.


(A) In human bodies, the gas bubbles released in this way often get trapped in the joints, causing extreme pain for the diver. This pain often makes it impossible for the diver to straighten up, which is why this condition is aptly named the bends. 2

(B) Helium is substituted for nitrogen in this mixture because it is less soluble than nitrogen and therefore less likely to dissolve in the bloodstream. This offers better protection against one of the major hazards of diving, called the "bends." 0

(C) If a diver returns to the surface too quickly after a dive, the relatively lower pressure at the surface than deep underwater causes dissolved gases to bubble out of solution in the blood. The effect is similar to the frothing in a bottle of soda water when the cap is removed. 1


① A-C-B ② B-A-C ③ B-C-A ④ C-A-B ⑤ C-B-A

 


86. 20수특 15-6

 

In England in the early 1900s property owners whose land was being eroded by wave action clamored for the Government to take preventive action.


(A) It must be admitted, however, that the land lost probably was good cliffland on the open coast which disappeared in a spectacular way, whereas the land gained was low, sandy and not particularly valuable. Non-geologists are usually not aware that the very existence of a cliff is warning that erosional processes are at work, even though the changes seem to be very slow. 2

(B) After making a careful survey, the commission reported that over a period of thirty-five years England and Wales lost 4,692 acres and gained 35,444 acres, giving a net gain of nearly nine hundred acres a year. This finding seemed to prove that people whose land disappeared complained more loudly than those whose land was increasing. 1

(C) Their island was disappearing beneath the sea! They argued so loudly that a Royal Commission was appointed to study the matter. 0


① A-C-B ② B-A-C ③ B-C-A ④ C-A-B ⑤ C-B-A

 


87. 20수특 15-7

 

Solid objects cohere as wholes.


(A) Rather, the applied force is converted by the liquid into an isotropic one that will urge the liquid to move in any direction open to it, unconstrained by any tendency for the liquid body to cohere. Water will tend to leak from a pipe with equal facility in any direction and not just in the direction of the weight of the head of water bearing down on it. 2

(B) While their shapes can be distorted to some degree to the extent that they are elastic, in the main they accelerate or tend to move as wholes in the direction of an applied force. Bodies of liquid differ in this respect. 0

(C) They freely adapt their shape to a containing vessel or an immersed solid and will simply give way to a solid object moving slowly through them. If a liquid body is subject to a force it will not tend to move as a whole in the direction of that force. 1


① A-C-B ② B-A-C ③ B-C-A ④ C-A-B ⑤ C-B-A

 


88. 20수특 15-8

 

Subjectivity is an integral part of all art, and sometimes, even where it seems the least likely, one finds a comprehensive communication between the artist and the audience very difficult.


(A) Therefore, an audience can never be sure exactly what the artist had in mind. This holds true on all levels, and thus, perfect communication cannot occur between most artists and their audiences through their art alone. 2

(B) No one mistakes that this painting is the portrait of a woman; that much we know. However, the intriguing smile in this painting is interpreted in so many different ways, in terms of what state of the mind this smile depicts. 1

(C) For example, not just in abstract painting, but in the most straightforward painting. Just take one of the best-known paintings, the Mona Lisa, painted by Leonardo da Vinci. 0


① A-C-B ② B-A-C ③ B-C-A ④ C-A-B ⑤ C-B-A

 


89. 20수특 16-01

 

Recent research has shown that trees are not as passive as we long supposed.


(A) However, when the scientists simply broke off new growth without applying any saliva, all the beeches and maples produced were hormones to heal the damage as quickly as possible. 2

(B) To investigate this further, European scientists simulated attacks on small beeches and maples. Whenever a roe deer takes a bite out of the top growth of a young tree, it leaves a little saliva behind in the wound, and it soon became clear that wounded trees can clearly detect the presence of this saliva. 0

(C) To simulate browsing by roe deer, the researchers cut off buds or leaves and dripped roe deer saliva onto the damaged areas. What they noticed was that in response the little trees produced salicylic acid, which in turn led to an increased production of bad-tasting defensive compounds, which discouraged the roe deer from eating them. 1


① A-C-B ② B-A-C ③ B-C-A ④ C-A-B ⑤ C-B-A

 


90. 20수특 16-02

 

Historical linguists study the languages spoken today, and from them make estimates about the ancestral languages from which they descended.


(A) In genetics a person has more and more ancestors as one goes to earlier generations, while a language has a single ancestor at each stage. The "tree model" of languages presents the range of languages descended from an ancestor, and indicates relationships with other languages descended from the same ancestor. Because of the single-ancestor characteristics of the linguistic "tree model," language gives more evidence on path of early human migration than does genetics, because it allows for fewer possibilities. 2

(B) Where possible, linguists also work from written records on languages in earlier times. For linguistics (as for genetics), we assume that present data give us the remnants of earlier communities. 0

(C) But the definition of "earlier community" is different in each case. For language, it is assumed that each language has one parent. 1


① A-C-B ② B-A-C ③ B-C-A ④ C-A-B ⑤ C-B-A

 


91. 20수특 16-03

 

The more things change, the more they stay the same.


(A) For them, what they see around them is "normal," and they have nothing to compare it with until they start to learn about history. It has always been this way. Parents feel the changes in the world; kids don't. 1

(B) Parents often react defensively, and children do not understand what all the fuss is about. Negative parental reactions often originate in hostility toward change. Most adults tend to see their own formative years as normal and what comes afterward as a decline. The only constant is change, and parents and their children experience this in fundamentally different ways. 2

(C) One thing that has remained the same over the centuries is parental disdain for the new kind of world they see around them. This world is only new to us, and it's the only one our children know. They haven't lived long enough to see the kind of social change that has taken place in the last twenty or thirty years. 0


① A-C-B ② B-A-C ③ B-C-A ④ C-A-B ⑤ C-B-A

 


92. 20수특 16-04

 

Because scientific research is so often conducted in the interests of national defense or under the sponsorship of private firms that hope to profit from applications of the findings, the norms of common ownership and publication are often suspended.


(A) The public laboratory had, or claimed to have, no such business interest. Eventually the competing teams compromised and issued a joint publication of the map, but the controversy and legal battles over issuing patents for genetic material have continued. 2

(B) Such situations have led to innumerable conflicts in scientific circles. An outstanding example is the race to publish the complete map and inventory of the human genome ― all the sequences of human DNA that constitute our genes. 0

(C) In 2001, two rival groups raced to be the first to complete the research, one in the private sector, the Celera Genomics Corporation, led by J. Craig Venter, and the other a government-funded laboratory headed by Francis Collins. An argument raged over whether the private corporation was trying to establish patents on human genetic sequences, a violation of the norm of openness. 1


① A-C-B ② B-A-C ③ B-C-A ④ C-A-B ⑤ C-B-A

 


93. 20수특 17-12

 

Looking for patterns works because our memories are organized in terms of what psychologists call 'schema'.


(A) A schema is a familiar pattern of relationships stored in your memory. That way they form memories so strongly linked that they are recalled more or less as a single unit. This is so powerful that it doesn't just influence your way of remembering lists, it actually affects your entire way of thinking. In one experiment, chess grandmasters and masters were tested against ordinary chess players to see how accurately they could remember the position of 20 to 25 chess pieces placed randomly on a board after glancing at the board for 5 to 10 seconds. 0

(B) And if you can reduce complex inputs to simple chunks, you'll find you can think about them much more clearly and effectively. As a general rule, if you ever find yourself forgetting anything, it is not because your brain is declining or your memory is receding. It's simply because you are not using the correct memory technique to help you store and retrieve the information.2

(C) The masters and ordinary players were pretty similar in being able to remember the places of only 6 pieces. Yet if the pieces were arranged in the form of a game (unknown to anyone), the grandmasters and masters could suddenly remember all the positions, while the ordinary players could still manage only 6. It was clear that this was not simply a memory feat 一it was due to the grandmasters' and masters' ability to see the positions as a single chunk or schema. It is clear that the more you develop schema, or patterns, the better you'll remember things. 1


① A-C-B ② B-A-C ③ B-C-A ④ C-A-B ⑤ C-B-A

 


94. 20수특 17-34

 

Novelty compels both humans and animals to engage with the unfamiliar.


(A) Indeed, our strong desire for novelty has evolutionary roots, improving our survival odds by keeping us alert to both friends and threats in our environment. As new parents quickly learn, when given a choice, babies consistently look at, listen to, and play with unfamiliar things. One of my favorite moments from early parenthood was when I watched my infant son notice his hands for the first time. His discovery stands out as a metaphor for learning:. 0

(B) That is, the people who traveled the farthest from home may have had some biological propensity to experience mysterious new places. And yet, while we are born with a strong drive to seek novelty, this drive fades over time. As we grow older, other desires take over, like wanting more predictability. The organizations we build and join reflect this reality: paychecks at the same time each week or month, evaluations according to established processes, jobs that involve a known set of activities. 2

(C) His interest in what those strange, wonderful appendages could do was his first step toward controlling them. The preference for novelty is an efficient way for immature cognitive systems to process information, helping babies cope with changes to their environment before releasing their inner explorer. Interestingly, in human genetics, a preference for novelty has been linked to the migration of early humans to the far reaches of the earth. Recent studies have shown that human groups that migrated the farthest from Africa had more of the genes linked to novelty seeking. 1


① A-C-B ② B-A-C ③ B-C-A ④ C-A-B ⑤ C-B-A

 


95. 20수특 17-56

 

You've no doubt heard the old saying: "when the going gets tough, the tough go shopping!"


(A) Starving E. coli colonies (hyper-)mutate at a rate one thousand times greater than that which would normally be the case in a well-fed colony. You can even find certain hypermutators within a normal or well-fed colony, sitting there, ready to spring into a hypermutational mode at the first sign of serious stress. As Frank T. Vertosick so humorously put it: "these hypermutators are the Van Goghs of the microbial world—somewhat insane, but infinitely creative". 2

(B) In a sporting team, it has been called throwing away the game plan, and this is exactly what hypermutation involves. When a bacterial colony is in a critical situation ― when survival is on the line ― something very strange happens — the bacteria suddenly start mutating at an extraordinarily rapid rate. In so doing, they are consulting the microbial lending library, hoping that they can come up with a mutation that might get them out of their crisis. 1

(C) That is precisely what bacteria do when they find themselves in deep trouble. They go shopping for useful genes that can help to get them out of the mess. There comes a time in the life of any organism or organization, when it has to try something completely different. 0


① A-C-B ② B-A-C ③ B-C-A ④ C-A-B ⑤ C-B-A

 


96. 20수특 17-78

 

Social sanctions vary in degree of formalization.


(A) In most stable associations there are highly formal procedures, such as ceremonies for honoring those whose services are believed to have contributed to the well-being of the membership and for the discredit or exclusion of those whose activities have been considered harmful. In our society, for example, there are courts of law and means of judging criminals which are so complex that only specialists can understand them. Some sociologists attach great importance to such highly formalized sanctions and have even defined the organized group as one in which the social structure is protected and reinforced through formal sanctions. Such norms are without doubt controls on deviant behavior, but for most people the less formal sanctions, the spontaneous displays of approval or disapproval, prove more effective. 0

(B) Since roles consist of reciprocating claims and obligations, they cannot be maintained without the cooperation of others in complementary roles. When a person speaks to a colleague, he ordinarily has a minimal claim upon him to respond in some way. Others may, however, refuse to live up to their obligation to be polite as a way of indicating their disapproval. 2

(C) Those who are about to violate some rule are often stopped short by the show of displeasure on the part of others. Ridicule and gossip are especially effective. In some cases deviant parties may be excluded informally, even when they continue to retain membership in the group. Among the most effective of the informal sanctions is the deprivation of mutual services, the refusal of others to honor the claims of the violator's role. 1


① A-C-B ② B-A-C ③ B-C-A ④ C-A-B ⑤ C-B-A

 


97. 20수특 18-13

 

Cuoi was born to a poor family and he had to work at various jobs to provide for his family.


(A) One day, while gathering wood in the forest far from home, Cuoi came upon a tiger cub and picked it up. Then, he heard a frightful growl. It was the mother tiger. Cuoi threw the cub to the ground and scrambled in terror up into the branches of the nearest tree. A moment later the tigress came crashing through the underbrush and growled as she saw the body of her dead offspring. Cuoi, in his haste to escape, had thrown the cub to the ground with such force it had been killed. Then, a strange thing happened. The tigress walked to a nearby stream and gathered the leaves from a banyan tree. She chewed them into a pulp which she then applied to the head of the dead cub. 0

(B) Cuoi's mother paid no attention to this warning and continued to dump dirty water near the tree. One day the tree began to slowly pull itself from the soil and to fly up into the sky. Returning from his chores, Cuoi noticed this and grasped its roots to keep it from flying away. But he was not heavy enough to keep the tree on the Earth. Instead, he was carried with it into the sky. After many days, Cuoi and the tree reached a strange new world — the Moon. Cuoi planted the tree there and sat down to figure out a way out of his predicament;. but there was no solution. There on the Moon he has sat waiting, year in and year out, even until today. 2

(C) Immediately the young tiger jumped to its feet and ran about as if nothing had happened. When the tigress and her cub had disappeared, Cuoi went to the miraculous banyan tree and gathered a handful of its leaves. On the way home he came upon a dead dog lying by the side of the road. Cuoi then chewed the leaves into a pulp and applied it to the dog's head. After a few minutes the animal was restored to life;. it jumped to its feet. Cuoi went back to the banyan tree, uprooted it, and replanted it in his yard. He warned his mother never to dump dirty water where the tree was planted. "Otherwise," he said jokingly, "it will fly away into the sky." 1


① A-C-B ② B-A-C ③ B-C-A ④ C-A-B ⑤ C-B-A

 


98. 20수특 18-46

 

The older man, a prominent baseball figure named Branch Rickey, suddenly changed his matter-of-fact, businesslike expression and turned on the young African-American athlete.


(A) Moments earlier they had been discussing the possibility of a career in big-time baseball for the young African-American man, Jackie Robinson. The year was 1945, and African Americans were barred from the major leagues. Rickey had warned Robinson that he would need two things to succeed. These were the ability to play baseball with the best, and the ability to stand the massive resentment toward the first African-American player in major-league baseball. In testing Jackie's patience in the latter, Rickey had tried to demonstrate, through pretended anger, the hate Robinson would meet. "Do you think," said Rickey, referring to his dramatic and realistic portrayal of a racist, "that you can get through this kind of thing?" "I can," replied Robinson softly, "if I pray." 1

(B) Rickey's prediction of trouble ahead could not have been more accurate. There were boos in the stadium, constant insults, and opposing players using steel spikes to hurt him. Teammates refused even to give him the traditional handshake and cheer when crossing the plate after a home run. But Jackie endured. And after his winning the Rookie of the Year Award in 1947, fans became aware of the prayer-born patience that made him one of the great men of the game. No one should have to qualify for sainthood in order to play baseball, but he humbled himself. Both he and Rickey were later honored by being elected into baseball's Hall of Fame. 2

(C) "You're just a black man," he shouted, "so you'll never succeed in the big leagues!" The young man, stiff with hurt and surprise, clenched and unclenched his fists threateningly, but the older man moved closer and kicked him with hostility. Then it was over. The older man — the baseball-team owner — stepped back, and the two men studied each other silently. The younger man spoke: "Why do you have to say these things to me, Mr. Rickey?" Branch Rickey, owner of the Brooklyn Dodgers and respected figure in baseball, dropped the false pose he had taken, smiled, and gently touched the young man's shoulder. 0


① A-C-B ② B-A-C ③ B-C-A ④ C-A-B ⑤ C-B-A

 


99. 20수특 18-79

 

The abbot woke up early one morning.


(A) Nothing was unusual in that. But this morning he was awakened by the sound of something moving in the nearby shrine room. That was unusual because most of his monks would normally be practicing their morning "chanting" so he went to investigate. In the darkness he saw a silhouette of a hooded figure. It was a burglar. After a few moments of silence the abbot said kindly, "What do you want, my friend?" "Give me the key to the donation box," said the burglar. The abbot saw a weapon in his hand but felt no fear. He felt only compassion for the young man. "Certainly," he said, slowly handing over the key. As the thief hurriedly emptied the box of cash, the abbot noticed the robber's jacket was torn, his face pale and gaunt. "When was the last time you have eaten, dear boy?" asked the abbot. 0

(B) He was sentenced to ten years in jail. Just over ten years later, the same abbot awoke to the sound of someone in the shrine room. He got up to investigate and, saw the old burglar standing next to the donation box. "Remember me?" shouted the burglar. "Yes" sighed the abbot. "Here's the key." Then the burglar smiled, and said gently, "Sir, put away the key. I couldn't stop thinking about you all those long days in prison. You were the only person in my entire life who was kind to me, who actually cared about me. I have come back to steal again, but this time I have come to take your secret of kindness and inner peace. Please, make me your disciple." 2

(C) "Shut up!" barked the burglar. The abbot nicely responded, "You'll find some food in the cupboard next to the donation box. Help yourself." The thief paused a moment in confusion. He was taken aback by the abbot's consideration for his welfare. Still, he hurriedly filled his pockets with cash from the donation box and food from the cupboard. "And don't call the cops," he shouted. "Why should I call the police?" answered the abbot calmly. "Those donations are to help poor people like you, and I have freely given you the food. You have stolen nothing." A few days later, the abbot read that the burglar had been caught robbing another house. 1


① A-C-B ② B-A-C ③ B-C-A ④ C-A-B ⑤ C-B-A

 


100. 20수특 18-1012

 

It is something of a common idea among animal behaviorists that wild animals do not tolerate disabilities, and that animals who are unfortunate enough to be born with a physical disability or fall ill rarely last very long.


(A) One foot was missing and one of her legs was several inches shorter than the other. Concerned on the first night that Helen might become the object of contempt from the other hens and roosters, Sturla set up a special nest in the barn. But when she opened up the door the following morning, a triumphant Helen greeted her sitting proudly on the top perch. Blind and lame, she had somehow found this spot. Far from feeling contempt for Helen's disabilities, the other birds stood in a kind of admiration for her, and she lives to this day in complete harmony with the rest of the flock, grooming her feathers, enjoying the sun, and dust-bathing with pure delight. 2

(B) I am doubtful. Recent research on many species has shown that young animals born with serious disabilities are nevertheless able to live with the help of their mothers and sometimes other friends and relatives. This is particularly true of elephants but applies to many species. Indeed, animals may have no concept of "disability" in the way humans do. 0

(C) Inspiring in this instance is the account Kim Sturla gave of Helen, a completely tame hen who was found wandering the streets of San Francisco. She was totally blind, and dogs were attacking her when a homeless person took pity on the hen and rescued her. She was taken to the city's animal shelter, where a call was put through to Animal Place to see if they would be willing to give her a home. Helen was born with a condition called cryptophthalmos, meaning that her eyelids had never formed properly and therefore never opened. 1


① A-C-B ② B-A-C ③ B-C-A ④ C-A-B ⑤ C-B-A

 


101. 20수특 19-1

 

William Allingham, poet and editor, is known mainly through a small group of poems that regularly appear in anthologies of Irish verse.


(A) In his own day, however, Allingham was well known to poets such as Robert Browning, Dante Gabriel Rossetti, and Alfred Tennyson, and his work inspired later Irish poets, including W. B. Yeats and John Hewitt. He was born in Ballyshannon, County Donegal, Ireland and his first job was in the local bank, where his father was manager. 0

(B) He befriended the influential poet, critic, and editor, Leigh Hunt, to whom he dedicated his first book of poems in 1850. His most ambitious work was the verse novel, Laurence Bloomfield in Ireland, in which he tried to promote reform of land ownership and tenants' rights. 2

(C) In 1846, he took a post with the national excise service and spent the next twenty-four years working as a customs officer. He first visited England in 1843 and eventually settled there in 1863. 1


① A-C-B ② B-A-C ③ B-C-A ④ C-A-B ⑤ C-B-A

 


102. 20수특 19-2

 

The first tunnel built under the Thames was designed by Marc Isambard Brunel (1769—1849).


(A) When it opened, it was at first targeted by thieves who would lie in wait in the shadows for unsuspecting pedestrians. The East London Railway took over the tunnel in 1869, and it now carries the East London Underground line across the river. 2

(B) Once released, Brunel worked on his observation, and eventually constructed a huge drill with corkscrew blades that passed dug-up material down its length as it moved forward, which was used to bore a tunnel from Wapping to Rotherhithe. The tunnel took 18 years to build, and 10 men died during its construction. 1

(C) Brunel fled the French Revolution for New York and then London, and at one point found himself in debtor's prison. While in prison, he watched a shipworm bore a hole through a piece of wood by passing the chewed-up wood pulp through its body and excreting it as it progressed. 0


① A-C-B ② B-A-C ③ B-C-A ④ C-A-B ⑤ C-B-A

 


103. 20수특 19-3

 

At Greenwich in 1796, the astronomer Maskelyne dismissed Kinnebrook, his assistant, because Kinnebrook observed the times of stellar transits almost a second later than he did.


(A) His attention was called to the "error," and it would seem that he must have striven to correct it. Nevertheless, it increased during the succeeding months until, in January, 1796, it had become about eight tenths of a second. 1

(B) T hen Maskelyne dismissed him. The error was serious, for upon such observations depended the calibration of the clock, and upon the clock depended all other observations of place and time. 2

(C) Maskelyne was convinced that all through 1794 there had been no discrepancy between the two of them. Then in August, 1795, Kinnebrook was found to be recording times about half a second later than Maskelyne. 0


① A-C-B ② B-A-C ③ B-C-A ④ C-A-B ⑤ C-B-A

 


104. 20수특 19-4

 

When her grandmother didn't answer right away, Kay got worried and started to say something, but before she could she watched as Teresa reached into the drawer by her bed and to Kay's astonishment, pulled out a tape recorder.


(A) I need you to listen closely." Teresa looked at Kay to make sure she was listening. Too shocked to say anything, Kay nodded as she sat down in the chair facing her grandmother's wheelchair as Teresa played the tape. 2

(B) What was her grandmother up to now? Where could she have gotten a tape recorder? And from the looks of it, she knew exactly how to use it. Kay had never seen her grandmother with anything like that. 0

(C) Teresa didn't even like using the phone. She once told Kay that she was allergic to the stuff the world made. She never said manmade. She always said "world made" so how and why ..., as the question was beginning to form Teresa said, "I made this recording so I won't forget anything. 1


① A-C-B ② B-A-C ③ B-C-A ④ C-A-B ⑤ C-B-A

 


105. 20수특 20-1

 

For over a million years humans lived in small, mobile groups gathering their food from the wild and hunting animals.


(A) When resources permitted, they came together in larger groups and occasionally, when they could rely on particularly rich sources of food, they became semi-sedentary. Then, in a relatively short period of time after the end of the last ice age, about 10,000 BCE, this stable and well-balanced way of life began to change. 0

(B) Hunting and gathering groups survived but they were increasingly pushed into more marginal areas and those that the farmers could not utilize. By the twentieth century only a few groups of such people survived to be studied by anthropologists. The adoption of farming was the most fundamental change in human history and led on to all that we call civilization and recorded human history. 2

(C) Across the world humans slowly began to settle in one location and replace gathered plants with ones grown on special plots of land while a few animals were also domesticated. In less than ten thousand years this new, agricultural way of life had spread around the globe. 1


① A-C-B ② B-A-C ③ B-C-A ④ C-A-B ⑤ C-B-A

 


106. 20수특 20-2

 

Most historians and philosophers agree that it was the teachings of the seventeenth-century French philosopher Rene Descartes that I ushered in the thinking of the modern age and began the unraveling of the ancient link between emotions and health.


(A) In Descartes's orderly division of the world into rational and irrational — provable and unprovable — emotions and their relationship to health and disease clearly fell into the latter domain. And there they remained until scientific tools powerful enough to challenge the categorization could rescue them. 2

(B) In his reaction to the religious wars and the resulting chaos that spread across Europe for most of his adult life, Descartes formulated the concepts of rationalism and the necessity of visible proof that were to become the founding principles of modern science. 0

(C) In that era, emotions seemed to be a thing of magic, fleeting and undefinable in the framework of the science of the day. 1


① A-C-B ② B-A-C ③ B-C-A ④ C-A-B ⑤ C-B-A

 


107. 20수특 20-3

 

Fashions and social pressures shift.


(A) Needless to say, this was not a widely pursued innovation, but enough affluent people did do so in their quest to maintain their position as societal leaders. As a result, the log house reasserted its position as an American icon, regardless of the ethnic background of its original builders. 2

(B) Throughout almost the first three-quarters of the 20th century, log houses in the United States, as in Norway, were considered to be rough, primitive, and low-class housing. As a consequence, weatherboards were widely used to mask earlier log construction. 0

(C) However, in the prosperous 1960s, when many individuals were seeking a challenge to the status quo, fashions changed and social pressure relaxed. These changes encouraged persons who wished to ride the crest of changing fashion, to seek out hidden log buildings, to remove the siding, and to enjoy the glow of their visual confirmation of society's rediscovery of its heritage. 1


① A-C-B ② B-A-C ③ B-C-A ④ C-A-B ⑤ C-B-A

 


108. 20수특 20-4

 

A suitable way to describe how geologists perceive rocks and landscapes is the metaphor of a palimpsest — the term used by medieval scholars to describe a parchment that was used more than once, with old ink scraped off to allow a new document to be inscribed.


(A) In the same way, everywhere on Earth, traces of earlier eras persist in the contours of landforms and the rocks beneath, even as new chapters are being written. The discipline of geology is similar to an optical device for seeing the Earth text in all its dimensions. 1

(B) Invariably, the erasure was imperfect, and traces of the earlier text survived. These remnants can be read using X-rays and various illumination techniques, and in some cases are the only sources of very ancient documents (including several of the most important writings of Archimedes).0

(C) To think geologically is to hold in the mind's eye what is not only visible at the surface but also present in the subsurface, what has been and will be. 2


① A-C-B ② B-A-C ③ B-C-A ④ C-A-B ⑤ C-B-A

 


109. 20수특 21-1

 

One possible disaster resulting from warmer temperatures in northern areas is a potential tipping point associated with the Greenland ice sheet.


(A) Large amounts of freshwater entering the North Atlantic off Greenland as the ice melts could shut down the conveyor belt because freshwater does not sink like saltwater. 1

(B) The "Atlantic conveyor belt" consists of warm water moving up the eastern coast of North America as the Gulf Stream and then sinking 10,000 feet near Greenland and moving back to the equator, where it warms and rises again. 0

(C) This could trigger an ice age in North America and Europe — an ironic consequence of global warming but one associated in the past with the conveyor-belt shutdown. Climate scientists worried about this possibility several decades ago and then decided about a decade ago that it was very unlikely, but they have now begun to worry about it again. 2


① A-C-B ② B-A-C ③ B-C-A ④ C-A-B ⑤ C-B-A

 


110. 20수특 21-2

 

Dams affect keystone species.


(A) First they block the migration of salmon upstream to their spawning streams. Scientists have created fish ladders as well as management schemes that transport the fish around the obstacles, but these efforts have limited success. As salmon are separated from the ecological functions of a habitat, other keystone species are impacted. 0

(B) If these species disappear, the animals that hunt them disappear. Rivers that do not produce a bonanza of salmon will not attract bears. They migrate to other areas entirely, creating a gap at the center of the ecological arch. 2

(C) In this example, one such keystone species would be the grizzly bear. Without a large amount of salmon, grizzly bears cannot transport the amount of oceanic nutrients inland to help enrich soils for trees. Lack of vegetation cover then affects songbirds and small mammals. 1


① A-C-B ② B-A-C ③ B-C-A ④ C-A-B ⑤ C-B-A

 


111. 20수특 21-3

 

Sixty-five million years ago a meteor the size of New York slammed into the Earth.


(A) The upper ocean became acidified for a brief moment, perhaps only one or two years. But that was enough. More or less every sea creature that built shells or skeletons out of calcium carbonate became either rare or extinct. 1

(B) A handful of corals must have survived, or we would not still have them on Earth today. But they were nonetheless too scarce to leave their imprint; they did not reappear in the fossil record for a full two million years. 2

(C) The environmental chaos that ensued is widely believed to have put paid to the dinosaurs. But it also had a less well-known effect. According to Ken Caldeira at the Carnegie Institution of Washington in California, the meteor also threw up vast amounts of sulfur, which then rained down on the ocean as sulfuric acid. 0


① A-C-B ② B-A-C ③ B-C-A ④ C-A-B ⑤ C-B-A

 


112. 20수특 21-4

 

It is not common for both sides of a noise barrier to look identical because it is usually only one side of the barrier that is required to reflect or absorb noise.


(A) The difference between the two sides of a barrier should be determined on aesthetic as well as acoustic grounds: each side of the barrier should be designed to integrate with the landscape character and the backdrop against which it is to be viewed. Thus, the face of the barrier that may be facing a road corridor may include a pattern or have a bright colour, whereas the other face, which could stand opposite housing, may be treated in a more discreet fashion. 0

(B) Planting on either side of the barrier should also be designed with each separate identity in mind. 2

(C) Here, the facade may be plain and designed to merge in with neighbouring planting. Most barrier types can be designed with this in mind with the obvious exceptions of transparent barriers and many bio-barriers. 1


① A-C-B ② B-A-C ③ B-C-A ④ C-A-B ⑤ C-B-A

 


113. 20수특 22-1

 

The people who came before you invented science because your natural way of understanding and explaining what you experience is terrible.


(A) Those data sit there, naked and exposed, so they can be reflected upon and rearranged by each new visitor. Scientists and laypeople will conjure up new stories using the data, and they will argue, but the data will not budge. They may not even make sense for a hundred years or more, but thanks to the scientific method, the stories, full of biases and fallacies, will crash against the facts and recede into history. 2

(B) You prefer easy-to-understand stories, and thus turn everything in life into a narrative so that complicated problems become easy. Scientists work to remove the narrative, to boil it away, leaving behind only the raw facts. 1

(C) When you have zero evidence, every assumption is basically equal. You prefer to see causes rather than effects, signals in the noise, patterns in the randomness. 0


① A-C-B ② B-A-C ③ B-C-A ④ C-A-B ⑤ C-B-A

 


114. 20수특 22-2

 

As the sun rises in the morning, sunlight warms the ground, and the ground warms the air in contact with it by conduction.


(A) Given their random motion, some air molecules will cross this boundary: The "hot" molecules below bring greater kinetic energy to the cooler air; the "cool" molecules above bring a deficit of energy to the hot surface air. However, on a windless day, this form of heat exchange is slow, and a substantial temperature difference usually exists just above the ground. 1

(B) However, air is such a poor heat conductor that this process only takes place within a few centimeters of the ground. As the sun rises higher in the sky, the air in contact with the ground becomes even warmer, and there exists a thermal boundary separating the hot surface air from the slightly cooler air above. 0

(C) This explains why runners on a clear, windless, summer afternoon may experience air temperatures of over 50'C (122'F) at their feet and only 32'C (90 'F) at their waist. 2


① A-C-B ② B-A-C ③ B-C-A ④ C-A-B ⑤ C-B-A

 


115. 20수특 22-3

 

The idea of using cold temperatures to extend the shelf life of food has been known for centuries.


(A) He caught a chill, which became pneumonia, and he died shortly thereafter while still at Highgate. He died a martyr to his science and sadly history does not record the fate of the world's first oven-ready frozen chicken. 2

(B) The great Francis Bacon, the early seventeenth-century polymath rather than the twentieth-century painter, is generally credited with inventing the frozen chicken. It was not all he did, but certainly the only thing relevant to refrigeration. 0

(C) In the early spring of 1626, while on the way to Highgate in North London, for reasons unrecorded, Bacon decided to buy a chicken whose organs had been removed and stuff it with snow, thus demonstrating that refrigeration was a remarkable way of keeping food fresh for longer. Unfortunately, as the whole escapade was a spontaneous experiment, Bacon must have been unsuitably dressed for the snow. 1


① A-C-B ② B-A-C ③ B-C-A ④ C-A-B ⑤ C-B-A

 


116. 20수특 22-4

 

Genetic diversity, the heritable diversity among individuals and populations within species, provides the basis for evolution.


(A) The vast majority of forest genetic diversity remains undescribed, especially in the tropics. Estimates of the number of tree species vary from 80,000 to 100,000, but fewer than 500 have been studied in any depth. 1

(B) Over millions of years it has enabled forests and trees to adapt to changing conditions. Some tree species have been domesticated, but the management of forest genetic resources mainly involves tree populations that have undergone little selection by humans. 0

(C) Until recently, studies of forest tree genetic resources have concentrated on the few species regarded as the most suitable for domestication for use in plantations and agroforestry systems to produce wood, fibre or fuel. The present and future potential of most tree species to adapt in response to novel climatic conditions or for genetic improvement for human use remains largely unknown. 2


① A-C-B ② B-A-C ③ B-C-A ④ C-A-B ⑤ C-B-A

 


117. 20수특 23-1

 

Today zoo managers recognize the need to provide a place for privacy for their charges.


(A) Most species need the opportunity to be free from the peering faces. The animal's 'flight distance' — the distance at which it will flee an approaching human — must be respected. Since animals spend much of their time foraging, wherever possible food must be hidden so that it is sought. 0

(B) And this would be as distasteful as it would be unnatural. There is no means of replicating the hunt for the large carnivores in captivity. 2

(C) It is unnatural to provide a wild animal with unearned food ― thereby depriving it of one of its predominant activities in the wild — so every effort must be made to replicate that activity. Unfortunately, that can never be done for the large carnivores. Even if, say, one were to introduce a live zebra into a lion's area there would be no hunt — merely a short chase. 1


① A-C-B ② B-A-C ③ B-C-A ④ C-A-B ⑤ C-B-A

 


118. 20수특 23-2

 

Cruise ships generally stay at ports of call between six and twelve hours, although you will find a number of itineraries with overnight port stays.


(A) One solution to this potential disadvantage is to select a cruise that begins and/or ends in a large city ― and this is the majority of Mediterranean cruises. You can then add days at either or both ends of your cruise to allow more time for in-depth sightseeing. 2

(B) This limited time allotment in port is fine for small towns and other "specialized" ports of call, such as ancient historic sites or resort islands, but it can present a problem when visiting larger cities. A pet peeve of mine, for example, is the cruise that has Rome as a day port of call. 0

(C) How can anyone do justice to Rome in a single day? Again, how much of a problem this is depends upon your expectations. 1


① A-C-B ② B-A-C ③ B-C-A ④ C-A-B ⑤ C-B-A

 


119. 20수특 23-34

 

Does an NBA all-star who is young, handsome, rich, and famous need to be told by his coach that he's playing well?


(A) Apparently, the answer is yes. In 1999, Chuck Daly was considered one of the best coaches in the NBA, and Penny Hardaway, one of Daly's players on the Orlando Magic, was one of the stars of the league. Hardaway hit a mild slump in his shooting and the team lost three in a row. Daly said little to him, so Hardaway concluded that the coach had lost confidence in him. 0

(B) He reminded him that he was a great player, told him to have fun, and invited him to come and talk to him any time he wanted. Hardaway broke out of his slump and the Magic went on a winning streak. Hardaway said later, "I'm not saying that was the main reason we haven't lost since, but it definitely helped. He let me know how important I was to this team." 2

(C) In an interview with The New York Times, Hardaway said he had probably jumped to the wrong conclusion from the coach's silence. "I know we make all this money and everything, and you wouldn't think some of us need reinforcement. But you want to hear from your coach that you're doing well, even at this level." Daly, known as a players' coach, solved the problem by having a private, heart-to-heart talk with Hardaway. 1


① A-C-B ② B-A-C ③ B-C-A ④ C-A-B ⑤ C-B-A

 


120. 20수특 24-1

 

Although tone color has a scientific explanation, its function in music is aesthetic.


(A) Even at the most advanced stages of accomplishment, achieving what is considered to be a beautiful tone is a criterion of a good performance. 2

(B) Music is an art of sound, and the quality of that sound has much to do with our response to it. Indeed, the concept of tonal beauty varies considerably in different periods, styles, and cultures. 0

(C) On the other hand, within a particular context, ideals of beauty may be quite firmly established and performers often pay extraordinary prices for instruments that can produce that ideal sound. But no instrument automatically produces a beautiful tone, so the finest violin will produce a rasping, scraping sound in the hands of a beginner. 1


① A-C-B ② B-A-C ③ B-C-A ④ C-A-B ⑤ C-B-A

 


121. 20수특 24-2

 

Although they were internally organized by machines ― cameras — early photographs resembled drawings and paintings because they depicted the world according to linear perspective.


(A) What was being represented remained unchanged. This does not diminish the camera's importance in defining an image. 1

(B) The camera obscura was popular with artists because it automatically modified a scene by compressing form and emphasizing tonal mass according to Western pictorial standards. The camera was not designed as a radical device to unleash a new way of seeing, but evolved to produce a predefined look that took into consideration formulas and procedures such as composition, angle and point of view, quality of light, and selection of subject matter. 0

(C) As with most inventions, unforeseen side effects create unintentional changes. As imagemakers became more sophisticated they routinely used specific cameras and lenses to shape an image, and knowledgeable viewers can often trace the connections between the camera/lens and the resulting picture. 2


① A-C-B ② B-A-C ③ B-C-A ④ C-A-B ⑤ C-B-A

 


122. 20수특 24-3

 

The effectiveness of green architecture depended upon the balance of on-site energy sources with building energy conservation.


(A) In colder climates, construction was focused on solar energy gain, heat retention using higher insulation values and double-glazing, and even movable insulation. In warmer climates attention was given to the solar control of the envelope, especially roofs and western facades, as well as to natural ventilation and daylighting. 1

(B) The more "conservative" the building envelope, insulation and the tightness of construction, the easier it was to match essential energy loads to the availability of on-site resources. The design motto was "conservation first!" 0

(C) Conservation conscious design was in sharp contrast to previous modernist tendencies of spatial generosity, complexity of form, expansive glazing areas, and reliance on abundant fossil fuels and mechanical systems for heating and cooling. 2


① A-C-B ② B-A-C ③ B-C-A ④ C-A-B ⑤ C-B-A

 


123. 20수특 24-4

 

Music tourism sites and attractions generally attract two kinds of visitors: those particularly drawn for whatever reason to the memory or music of a particular performer, composer or genre (most obvious for festivals), and those who are there because the place fits into an itinerary devised for other reasons or because the visit is likely to be enjoyable.


(A) For French tourists, travel to New Orleans in part allows an experience of Francophone identities in the New World, just as it is more likely to be Americans who visit Jim Morrison's grave in Paris. 2

(B) Thus, at Abbey Road, a place identified in many general guidebooks, most visitors were there because they had some appreciation of the Beatles' music, but others were there simply because it was part of an agenda that included a range of obvious London landmarks. Particular links to music sites are, however, much more idiosyncratic and dependent on personal musical taste. 0

(C) Wider cultural and national links may also be evident. New Orleans and the Cajun region are popular with French tourists. 1


① A-C-B ② B-A-C ③ B-C-A ④ C-A-B ⑤ C-B-A

 


124. 20수특 25-1

 

Making a discovery, whether in the laboratory or library archives, about something that no one else in the world knows at a particular time is a thrilling experience for an undergraduate student.


(A) Nearly every U.S. college and university includes undergraduate research experiences in the learning opportunities offered to students. Institutions of higher education of all types are implementing a number of creative approaches to facilitate innovation, problem-solving and discovery, and they see undergraduate research as a critical vehicle for achieving these objectives. 2

(B) They may identify a new bacterial strain, make a connection between previously separate concepts, or uncover long-forgotten archival documents that lead to a new understanding of an issue. 1

(C) College students who have the opportunity to participate in undergraduate research can experience this thrill of discovery. 0


① A-C-B ② B-A-C ③ B-C-A ④ C-A-B ⑤ C-B-A

 


125. 20수특 25-2

 

A professor sent his sociology class to a school in a slum to record case histories of 200 young boys.


(A) Of the 180 still surviving in that neighborhood, all of these men had achieved extraordinary success as doctors, lawyers, and productive members of the community. The professor, determined to find out how these men had beaten seemingly overwhelming odds, followed up with interviews. 1

(B) The sociology students concluded that each boy appeared doomed to a bleak future. More than two decades later, another sociology professor sent his class out to find out what had happened to these 200 boys. 0

(C) Each of the men credited the same particular teacher; someone who had seen great potential in each of them. This teacher's expectations, held with conviction, drove her to actively nurture and support imminent success in all her students. 2


① A-C-B ② B-A-C ③ B-C-A ④ C-A-B ⑤ C-B-A

 


126. 20수특 25-3

 

Samuel and Pearl Oliner found large differences between European Gentiles who harbored Jews from the Nazis and those who did not: Rescuers reported close childhood associations with more people of different social classes and religions.


(A) All this suggests a piece of advice for prospective parents who want their children to develop a broadly charitable nature: Give them positive contact in the home with individuals from a wide spectrum of backgrounds. 2

(B) Not only was this expanded sense of "we" related to their decisions to aid people different from themselves during the war but also, when interviewed half a century later, rescuers were still helping a greater variety of people and causes. 1

(C) Moreover, while growing up, they felt a sense of similarity to a wider and more varied group of people than did nonrescuers. 0


① A-C-B ② B-A-C ③ B-C-A ④ C-A-B ⑤ C-B-A

 


127. 20수특 25-4

 

Have you ever said something to your child like the following?


(A) Your room is a mess; I want you to clean it up before you go to the game. Have you then wondered why she didn't do what she was told to do, right away and with a smile? 0

(B) This attitude, however, fails to consider the child's point of view. When you neglect to consider your child's thoughts, feelings, needs, and possible solutions to getting the room cleaned, you do so at the risk of losing her respect and goodwill. Your child's grumbling resistance is, in effect, a natural consequence of your choice to operate without her input. 2

(C) You made a unilateral decision, and she was expected to carry it out according to your time frame and standards. Because, After all, I'm the parent! 1


① A-C-B ② B-A-C ③ B-C-A ④ C-A-B ⑤ C-B-A

 


128. 20수특 26-1

 

The ancient Greeks had a word that lies at the heart of procrastination: akrasia, which means doing something against our own better judgment.


(A) The pile of important papers you never quite get to. The new job you put off looking for even though going to your current place of employment is the bane of your existence. 1

(B) When we procrastinate, we act against our own self-interests, satisfying the desire for immediate gratification by sacrificing our own longer-term goals and well-being. The essence of procrastination is to willingly put off something even though you expect the delay to make you worse off. 0

(C) Whenever we knowingly delay doing what's necessary in favor of the easier, less important task, we feed the demon of procrastination. 2


① A-C-B ② B-A-C ③ B-C-A ④ C-A-B ⑤ C-B-A

 


129. 20수특 26-2

 

In addition to gender, cultures have many other ways of dividing people into categories.


(A) Rituals, ceremonial occasions marking significant events such as weddings, graduations, and important holidays, are often events which are dressed for, where categories considered important by a society are visually displayed for all to see and appreciate. 2

(B) Common examples include age, class, religiosity (level of religious commitment), political stance, and value system. In all these examples, dress functions as a way of marking that a person is a member of a particular category. 0

(C) For example, older women in both the United States and Europe have created Red Hat Societies, in which red hats and purple clothing are used to formally mark entrance into middle and older age with fun and spirited apparel. These category systems help people living within cultural groups to interact easily, with dress functioning as a sign system allowing for rapid sorting of people into known and accepted groupings important to the given culture. 1


① A-C-B ② B-A-C ③ B-C-A ④ C-A-B ⑤ C-B-A

 


130. 20수특 26-3

 

James Banks states that "worldwide immigration is increasing racial, ethnic, cultural, linguistic, and religious diversity throughout the United States as well as in other Western nations such as the United Kingdom, France, Germany, and Australia."


(A) Christine Bennett adds to this argument by stating currently, more than ever before, there is an urgent need for citizens in the world that have a strong "multicultural competence" and who aim for global goals such as social justice and economic equality in order for there to be a sustainable peace. The intensified movement of people in recent times has not gone unnoticed, and must be addressed if peaceful societies are to exist. 2

(B) Banks argues that this is a time when the world "demands leaders, educators, and classroom teachers who can bridge impermeable cultural, ethnic, and religious borders, envision new possibilities, invent novel paradigms, and engage in personal transformation and visionary action." 1

(C) Indeed, these are not the only countries that are experiencing changing populations. 0


① A-C-B ② B-A-C ③ B-C-A ④ C-A-B ⑤ C-B-A

 


131. 20수특 26-4

 

Literacy is a common end of persons nowadays but reading and writing are recent inventions, only five thousand years old.


(A) It couldn't have. We were not literate for almost the entire history of our species. Furthermore, literacy does not seem to contribute to fitness, since there is an inverse correlation between fitness — as measured by birthrate, a proxy for inclusive fitness — and literacy. 1

(B) The birthrate is at zero or below in many parts of Europe where literacy is almost universal, and over 5 percent in places like Yemen and Niger, where literacy is low. Nonetheless, literacy seems to be really important for something else: human flourishing. 2

(C) There was no selection for literacy. In order to read, we utilize brain areas originally selected to track animals. One way to put the matter is that literacy didn't initially matter one iota for fitness. 0


① A-C-B ② B-A-C ③ B-C-A ④ C-A-B ⑤ C-B-A

 


132. 20수특 27-1

 

Reformers have seen the Internet as a means of moving toward the ideal of responsive democratic governance in the area of public administration.


(A) Just as the Internet has brought dramatic changes to e-commerce, so advocates of e-government, the provision of governmental information and services online, see the Internet as a way to modernize the public sector and democratize the relationship between individual citizens and their government. 2

(B) Information can be accessed and services can be delivered at the convenience of the citizen. No standing in line at a government office, no trying to reach a bureaucrat by telephone, no waiting for forms to arrive in the mail. 1

(C) They view the burgeoning presence of government on the Internet as an attempt to respond to citizens as clients and customers. They believe that conducting public business online provides many of the same advantages as conducting private business online. 0


① A-C-B ② B-A-C ③ B-C-A ④ C-A-B ⑤ C-B-A

 


133. 20수특 27-2

 

Even if we give robots the ability to learn what we want, an important question remains that AI alone won't be able to answer.


(A) How to combine these people's values when they might be in conflict is an important problem we need to solve. AI research can give us the tools to combine values in any way we decide but can't make the necessary decision for us. 2

(B) We can make robots try to align with a person's internal values, but there's more than one person involved here. 0

(C) The robot has an end user (or perhaps a few, like a personal robot caring for a family, a car driving a few passengers to different destinations, or an office assistant for an entire team); it has a designer (or perhaps a few); and it interacts with society — the autonomous car shares the road with pedestrians, human-driven vehicles, and other autonomous cars. 1


① A-C-B ② B-A-C ③ B-C-A ④ C-A-B ⑤ C-B-A

 


134. 20수특 27-3

 

Children, of course, have less control over their attention than adults, but when they do pay attention, they open their minds more fully to the messages presented.


(A) That, in turn, helped drive sales of toys, tickets to Disney's feature films, trips to Disneyland, and so on. And by the late 1950s, toy makers began creating toys specifically designed to be advertised on television — like Mattel's first girl-directed toy, the Barbie doll, whose commercials, which ran during breaks in The Mickey Mouse Club, chronicled glamorous episodes in Barbie's life. 2

(B) By the early 1950s advertisers had come to understand the commercial potential of reaching children by television. The Howdy Doody Show, featuring a clown and a dancing puppet, for instance, was sponsored by a famous food company, and during every show Howdy Doody would dance around a cereal box. 0

(C) But those were the early days. By the late 1950s, programmers had learned to create shows that in a sense served as advertisements themselves, like The Mickey Mouse Club, which helped nurture enduring attachments to characters like Mickey Mouse, Donald Duck, and other friends. 1


① A-C-B ② B-A-C ③ B-C-A ④ C-A-B ⑤ C-B-A

 


135. 20수특 27-4

 

With so many people sharing the most intimate details of their lives with the world, something was bound to disrupt the trajectory of online sharing.


(A) In this way users can control their digital footprints. 2

(B) The year 2013 saw NSA (National Security Agency) leaks, hackers targeting consumer credit cards, and blanket inquiries into individuals' personal lives through their online connections, to name a few. These invasions of privacy and more have inspired whole new platforms based on giving the user a digital experience that can be anonymous, deleted, and secure. 0

(C) For example, app developers have created a photo messaging app that enables users to send a photo or video with text to a specific group of people and control the time limit for how long they can view the sent message from one to ten seconds. When the time limit ends, the message is no longer available and is deleted from the app's servers. 1


① A-C-B ② B-A-C ③ B-C-A ④ C-A-B ⑤ C-B-A

 


136. 20수특 28-1

 

One approach to social facilitation that proposes an influence in social presence is based on the idea that I people generally try to present the best possible appearance to others and to make a favorable impression.


(A) This being the case, observers or coactors may not only motivate individuals to work hard at whatever task is being carried out, but also increase the person's sense of embarrassment when performance leads to failure. 0

(B) Difficult tasks are often failed, however, at least at the beginning. Embarrassment caused by such failure may cause stress and cognitive interference of sufficient intensity to disrupt performance. 2

(C) Failure is not likely to happen when the task is a simple or familiar one, so that the increased motivation is sufficient to produce improvement. 1


① A-C-B ② B-A-C ③ B-C-A ④ C-A-B ⑤ C-B-A

 


137. 20수특 28-2

 

Name a person who's accomplished something great, and you'll find a trail of risk along their personal path to greatness.


(A) If you're not failing, you're probably not stretching yourself. The comfort zone is a place where dreams go to die, and where vitality goes to die with it.2

(B) They weren't afraid to turn the volume up on their lives so they could ultimately turn the volume up on their love. When you're going after anything worthwhile in life, know that you're going to eventually fall, look stupid, and make mistakes;. it's part of the process of achieving. 1

(C) Do you think Martin Luther King, Jr., wasn't afraid? Do you suppose that JFK, Mother Teresa, Gandhi, and Nelson Mandela weren't scared about how their message would be received? These were people who heard the call, felt the fear, and moved forward regardless. 0


① A-C-B ② B-A-C ③ B-C-A ④ C-A-B ⑤ C-B-A

 


138. 20수특 28-3

 

We so often reach the conclusion, soon after we wake up, that "it's just one of those days," and by doing so create one of those days; or we enter a situation and predict that it's going to be bad — boring, irritating, frustrating, or annoying — and our prediction becomes a self-fulfilling prophecy.


(A) And while I should certainly allow myself to experience painful emotions at times, in some situations it may be appropriate to "fake it till I make it" and imbue myself and others with positive energy. 2

(B) Just as I am affected by other people's moods, other people are affected by mine. If I choose to enter a room with a sense of joy and excitement, my positive mood will spread and will affect those present. 1

(C) In fact, however, we can make most days and most encounters cheerful, lively, positive, and pleasant — if we choose to bring these kinds of emotions to the situation. Emotions are contagious. 0


① A-C-B ② B-A-C ③ B-C-A ④ C-A-B ⑤ C-B-A

 


139. 20수특 28-4

 

You can steer a conversation in any direction that you choose.


(A) what response might you get? She would tell you about its quality and the craftsmanship that went into it. When you say it's expensive, she'll talk about the quality. If you say that it looks beautiful, she'll tell you about the cost. By asking the right questions, you can steer the conversation in any direction you want and elicit the information that you need. 2

(B) Take this example. Let's say that while you are at a friend's house, she shows you her brand-new dining room table. If you want to know if it was really expensive, would asking directly be your best bet? Usually not, because she may get a little defensive. 0

(C) But if you said to your friend that it's the most gorgeous table you have ever seen, what might she respond with? You guessed it — how expensive it was! If you said, "This looks like it cost a fortune. How could you spend so much on a table?" 1


① A-C-B ② B-A-C ③ B-C-A ④ C-A-B ⑤ C-B-A

 


140. 20수특 29-1

 

In the United States, the coming of the railroads opened up local and regional markets to competition.


(A) Local producers had difficulty competing with large manufacturers, and many local companies went out of business or merged with other firms producing similar foods. 0

(B) In 1860, New York City had more than 2,000 butchers; only a few hundred survived in 1900. These butchers had counted on their customers' appreciation of the quality and flavor of their meats, but shoppers readily gave up locally butchered beef and pork for cheaper cuts shipped from the Midwest. 2

(C) The shipment of dressed beef from the Midwest to eastern cities, for instance, all but eliminated local slaughterhouses and reduced the number of butchers. 1


① A-C-B ② B-A-C ③ B-C-A ④ C-A-B ⑤ C-B-A

 


141. 20수특 29-2

 

There are limits to the idea of "timeless" fashion.


(A) Perhaps, then, being cool is also a matter of having freedom to exercise one's tastes, and the resources to be able to do this. 2

(B) It's hard to imagine a woman dressed in Jane Austen-era clothes as "cool" even though she might be fashionable given the standards of the time. Historical figures who might qualify as "cool," such as Lord Byron, are individuals who had a degree of financial and/or social freedom (usually arising from wealth and leisure) that allowed them to exercise their aesthetic choices in a way that was not possible for individuals who were more constrained by convention and circumstance. 1

(C) Fashions from the Victorian era or from Elizabethan times do not appear cool or fashionable now, no matter who wears them. Indeed, it is interesting to consider whether the concept of "cool" can be applied to historical eras in which fashion is constrained by social and gender conventions to a far greater degree than is the case in most modern liberal societies. 0


① A-C-B ② B-A-C ③ B-C-A ④ C-A-B ⑤ C-B-A

 


142. 20수특 29-3

 

Politicians, especially those in the national spotlight, are often jokingly accused of being narcissists but, in all seriousness, their profession lends itself to this particularly destructive personality trait.


(A) Finally, you constantly have people relying on you, believing in you, and holding you responsible as the sole representative of a cause. All of this power can lead to an exaggerated sense of self-importance that can cause some individuals to believe that the world revolves around them. 1

(B) That's when their out-of-control behaviors become easier to rationalize. 2

(C) For example, in order to be a successful candidate, you have to be unnaturally optimistic even in the face of probable defeat and possess high levels of self-esteem despite the constant criticism that comes with the territory. Furthermore, you are constantly given sole credit for successes — even though those successes were achieved, in part, by the work of many aides and assistants. 0


① A-C-B ② B-A-C ③ B-C-A ④ C-A-B ⑤ C-B-A

 


143. 20수특 29-4

 

Many of those who oppose globalization reserve their highest loyalties to the sovereign state, which they believe exists to protect their interests.


(A) Thus, anti-globalizers argue, globalization has created a democratic deficit by empowering institutions in which people have no voice and unleashing economic and cultural forces over which they have no control. 1

(B) Globalization, they believe, is eroding the rights and capacity of people to determine their own future. The result is alienation and anxiety, as people's lives are troubled by remote forces beyond their control or understanding. 2

(C) They argue that in democratic states, such as those in Europe and North America, citizens have a voice in determining their own fates but have little or no voice in the boardrooms of giant transnational corporations, remote international bureaucracies like the EU or WTO, or economic markets, and such institutions are not accountable to citizens. 0


① A-C-B ② B-A-C ③ B-C-A ④ C-A-B ⑤ C-B-A

 


144. 20수특 30-1

 

A middle-aged woman sat in a clinic's sitting room waiting to be fitted for a prosthesis to replace the leg she lost in an accident.


(A) The word "pirate" transformed her immediately and changed her entire outlook. She vividly imagined the romantic thrill of sailing the high seas in search of adventure. Some time later, the little boy asked what happened to her leg. She replied proudly "I' m a pirate too." 2

(B) She struck up a casual conversation and eventually asked what happened to his eye. He replied, "Oh nothing. I'm a pirate." 1

(C) Although considerable time had passed since the accident and she had adjusted well, she continued to feel emotionally devastated by her loss. The woman watched as a young boy with a patch over his eye played with some toys. The thought of losing an eye at such a tender age made her own difficult situation seem minor by comparison. 0


① A-C-B ② B-A-C ③ B-C-A ④ C-A-B ⑤ C-B-A

 


145. 20수특 30-2

 

The definition of a calorie does not refer to something that produces fat, but to something that produces energy.


(A) Calories are an important source of energy for the body and the brain. They are necessary for the growth of nails and hair and to replace skin cells. 0

(B) Consuming enough calories is necessary not only for healthy physical functioning but also for psychological well-being. The intake of sufficient calories is a necessary condition to be in a positive frame of mind. 2

(C) Calories produce the energy needed for walking, bicycling, reading, talking over the phone, texting and so on. Without the intake of calories, the body cannot develop energy for physical, psychological and social activities and becomes exhausted. 1


① A-C-B ② B-A-C ③ B-C-A ④ C-A-B ⑤ C-B-A

 


146. 20수특 30-3

 

The anti-aging industry promotes a particular image of longevity that is reduced to biomechanical processes at the cellular level.


(A) Gaining broader cultural knowledge of medicinal foods may offer alternatives to anti-aging discourse in which bodies are subject to biomechanical processes. Traditional systems of medicine offered insights based on observation of the subtle interactions of food and environment of human bodies. 1

(B) Such discourse focuses on longevity as an individual body's battle against aging rather than considering aging in a broader social and historical context. In an era when care of the self comprises a vast industry, healing through nutrition and healthy diets may seem too low-tech or slow. 0

(C) We can greatly influence our well-being through diet and nutritional knowledge, not just consuming dietary supplements. Longevity is not guaranteed, but the possibility of accessible self-managed care on a daily basis through attention to one's food can enhance the quality of one's life. 2


① A-C-B ② B-A-C ③ B-C-A ④ C-A-B ⑤ C-B-A

 


147. 20수특 30-4

 

As early as 1907, one prominent Russian zoologist, Ilya Metchnikoff, began to question if the "all bacteria are bad" orthodoxy might be flawed.


(A) He observed that the oldest villagers were drinking fermented yogurt containing the bacteria Lactobacillus bulgaricus. Metchnikoff suggested that one secret to longevity is consuming healthy bacteria. History would prove him right. 2

(B) He was also struck by the fact that some people lived to a ripe old age despite harsh rural conditions and poor hygiene associated with poverty. In Bulgaria, he noted, there were peasants in the Caucasus Mountains who lived beyond one hundred years. 1

(C) During the cholera epidemic of 1892 in France, Metchnikoff mixed bacteria together in a petri dish and found that some bacteria could stimulate cholera growth, but to his surprise found that other bacteria hindered it. This led him to speculate whether swallowing some types of helpful bacteria might be useful for preventing deadly diseases. 0


① A-C-B ② B-A-C ③ B-C-A ④ C-A-B ⑤ C-B-A

 


148. 20수특 T1-1

 

We received your application for the Steven Christopher Scholarship.


(A) This experience will greatly benefit your qualifications the next time you apply for the same scholarship. We encourage you to reapply for next year's slot, and wish you the best of luck in your academic career. 2

(B) However, we regret to inform you that the scholarship has been granted to someone else who has a wider experience in the industry. Considering that you are a few years younger than the others who are also applying, we encourage you to gain more experience and expose yourself more to different challenges and situations in the corporate world. 1

(C) We sincerely appreciate your interest. Each year hundreds of students apply for the scholarship, and your outstanding academic record placed you among the finalists. 0


① A-C-B ② B-A-C ③ B-C-A ④ C-A-B ⑤ C-B-A

 


149. 20수특 T1-2

 

I signaled to David, my diving teacher, that I was going down into the sea.


(A) I couldn't see David, I couldn't see the bottom, and I could barely see the surface. I was breathing too fast, and my suit was too tight and confining. My panic grew at a rate almost too fast to control. 1

(B) I worked to calm myself: Slow your breathing, adjust your buoyancy, move slowly. It worked; I didn't dash to the surface, I looked at David, and out of habit only I gave him a grin and a thumbs-up, but it was enough to refocus and follow my training, adjust my buoyancy, take a slow breath, and proceed down. Now I was looking at my old invertebrate friends on the cave wall that had been the site of my very first underwater job. 2

(C) As I slowly sank away from the lighted sea surface, I swam toward the cave wall. With a growing uneasiness, I realized that the cold was not nearly as much a problem as the dark. Underwater, it was completely, disorientingly dark. 0


① A-C-B ② B-A-C ③ B-C-A ④ C-A-B ⑤ C-B-A

 


150. 20수특 T1-3

 

People are distracted.


(A) But only multichannel information can somehow guarantee that a piece of information really reaches the audience. So, do not hesitate to repeat your content in emails, intranet, staff meetings and so on. At some point it will reach the minds of your people and stick in their memory. 2

(B) They are distracted because information is everywhere and the selection of the relevant messages becomes increasingly difficult. So, even if a piece of information was successfully delivered, it doesn't mean it's been noticed, understood, internalised and pondered. It may just be forgotten or neglected. 0

(C) This is why Consul Marcus Tullius Cicero said 'Repetita iuvant' (repetitions help). And the purpose is clear: ensuring the reception of that message or piece of information. It's best to repeat messages in different channels, even if it seems a redundant exercise. 1


① A-C-B ② B-A-C ③ B-C-A ④ C-A-B ⑤ C-B-A

 


151. 20수특 T1-4

 

The foundation of an effective team is the recognition that each member needs every other member, and no individual can be successful without the cooperation of others.


(A) As a young boy I was a very enthusiastic baseball fan. My favorite player was the Hall of Fame pitcher Robin Roberts of the Philadelphia Phillies. During the early 1950s, his fastball dominated the National League. 0

(B) I told my uncle that I knew how to stop Robin Roberts:. "Make me his catcher." 2

(C) My uncle, who took me to my first ball game, explained that opposing batters were so intimidated by Roberts's fastball that they were automatic "outs" even before they got to the plate. My uncle claimed that Robin Roberts was unstoppable. Even as a young boy I intuitively knew that no one was unstoppable by himself. 1


① A-C-B ② B-A-C ③ B-C-A ④ C-A-B ⑤ C-B-A

 


152. 20수특 T1-5

 

People or firms that purchase services come in contact with other consumers as well as the service employees.


(A) The two groups are significantly different in behavior, and the expectation is that they would not mix well within the facilities at the same time. Similarly, restaurants separate smokers and nonsmokers, and they should try to separate other patrons that show some potential for conflict. 2

(B) For example, a hotel guest waits in line at the front desk or the concierge desk with other guests. In addition, the guests share facilities such as the pool, the restaurant, and the fitness center. 0

(C) Therefore, service firms must manage consumer interactions to the best of their abilities to ensure customer satisfaction. For example, a hotel's sales office would not want to book group business with a nondrinking religious group at the same time as a reunion of military veterans. 1


① A-C-B ② B-A-C ③ B-C-A ④ C-A-B ⑤ C-B-A

 


153. 20수특 T1-6

 

Great coaches know that they can get their athletes to perform well by drumming certain ideas into their heads.


(A) This is basically the technique Jack Nicklaus, perhaps the greatest golfer of the past several decades, used to enhance his performance. Before every shot, he formed a mental picture in which he saw three things: (1) the target area the ball would land in, (2) the flight path of the ball to the target area, and (3) himself using the appropriate swing for that particular shot. 1

(B) Foremost is the idea that the players are winners, so that they will think only of winning and never about the possibility of losing. Chances are high that they indeed will win because the image of themselves as winners will force them to concentrate only on the moves that ensure winning. 0

(C) In short, if athletes define themselves as winners, they are more likely to win. By the same token, if athletes define themselves as losers, they will very likely lose. 2


① A-C-B ② B-A-C ③ B-C-A ④ C-A-B ⑤ C-B-A

 


154. 20수특 T1-7

 

A lichen is an organism consisting of a fungus and an alga living together, usually in an interdependent relationship.


(A) In contrast, areas with clean air can support larger varieties of lichens. Some lichen species are sensitive to specific air-polluting chemicals. Old man's beard and yellow Evemia lichens, for example, can sicken and die in the presence of excessive sulfur dioxide (S02), even if the pollutant originates far away. 1

(B) These hardy species are good biological indicators of air pollution because they continually absorb air as a source of nourishment. A highly polluted area around an industrial plant might have only gray-green crusty lichens or none at all. An area with moderate air pollution might support only orange crusty lichens. 0

(C) For this reason, scientists discovered S02 pollution on Isle Royale, Michigan, in Lake Superior, an island where no car or tall factory chimney has ever existed. They used Evemia lichens to point the finger northwest toward coalburning facilities in and around the Canadian city of Thunder Bay, Ontario. 2


① A-C-B ② B-A-C ③ B-C-A ④ C-A-B ⑤ C-B-A

 


155. 20수특 T1-8

 

Belize is the second smallest country in Central America in landmass, the smallest in population, yet is one of the most diverse in several ways.


(A) A coral barrier reef, the world's second longest, protects the coast from high sea tides, but not from frequent destructive hurricanes. The northern region is also lowland and heavily forested with old-growth hardwoods such as mahogany. These have been harvested for many years and have been important in the Belizean economy. 1

(B) Subtropical in climate with a wet and a dry season, the country is located on the Caribbean Sea with Mexico to the north and Guatemala to the south and west. Most of Belize is low-lying country. The long coastline is often swampy with lagoons and many small islands. 0

(C) For such a small country, there is surprising biodiversity. Belize is recognized as one of the world's ecological treasures, and as a result, ecotourism is a growing part of their economy. 2


① A-C-B ② B-A-C ③ B-C-A ④ C-A-B ⑤ C-B-A

 


156. 20수특 T1-9

 

The above table, provided by Modesto Junior College in Modesto, California, shows enrollment counts and success rates of the students who took the English basic skills course in two terms, the fall of 2014 and 2015, by their ethnic group.


(A) The number of Hispanic students accounted for more than 50% of the students who enrolled in the English basic skills course in both terms. Pacific Islander students had the highest success rate in both years while the students of Multi-Ethnicity had the lowest success rate in 2014 and 2015. 1

(B) In three ethnic groups, the enrollment count had fewer students in the fall of 2015 than that of 2014. 2

(C) Both the total number of enrolled students and their success rate in the fall of 2015 were higher than in the fall of 2014. Nearly all of the ethnic groups recorded higher success rates in 2015 than their previous year's success rates: only one ethnic group, Multi-Ethnicity, didn't have an increase in their success rate. 0


① A-C-B ② B-A-C ③ B-C-A ④ C-A-B ⑤ C-B-A

 


157. 20수특 T1-10

 

2020 Caroline County Art Show & Contest The 2020 Caroline County Art Show & Contest is just around the corner.


(A) ■ You must be 18 years and older to enter the contest. ■ The artwork will be judged by a guest art curator. 1

(B) Come and visit to view art created by local artists, or register your own work in the art contest! Art contest guidelines ■ Drop off your artwork at Pine Tree Art Gallery by December 27. 0

(C) Opening reception: Friday, January 3, 6-8 p.m. ■ General public ― $20 ■ Gallery members ― $10 ■ Participating artists ― Free Guest curator talk: Saturday, January 4, 10-11 a.m. ■ This year's guest curator is John Raymond. ■ No charge and open to all! For more information about the art show & contest, click here. 2


① A-C-B ② B-A-C ③ B-C-A ④ C-A-B ⑤ C-B-A

 


158. 20수특 T1-11

 

Automated External Defibrillators Automated External Defibrillators (AEDs) are portable devices that can detect life-threatening cardiac conditions and apply electrical shocks to the heart to restore heart rhythm, allowing blood and oxygen to flow to the heart, brain and body.


(A) 3- The machine will analyze the heart rhythm. 4- If an abnormal rhythm is detected, the AED will charge and instruct you to push the shock button. 5- Do not touch the victim or their clothing while the AED is analyzing or delivering a shock. 1

(B) 6- After the AED delivers a shock, immediately begin chest compression with your hands for two minutes. 7- Check the victim and repeat the analysis, shock, and chest compression as needed until medical professionals arrive. 2

(C) AEDs require no training, which means anyone nearby can provide treatment during cardiac arrest in those critical minutes before medical professionals arrive. How to use an AED 1- Place the AED near the victim's head, power on the unit, and wait for vocal instructions. 2- Apply the sticky pads to the victim's chest. 0


① A-C-B ② B-A-C ③ B-C-A ④ C-A-B ⑤ C-B-A

 


159. 20수특 T1-12

 

We behave (perform roles) as we think the role or roles should be performed (eg, daughter or son, mother or father, etc); this is known as role-taking.


(A) If someone smiles back at you, your assumption is that he or she has the same intentions as you ― to make your acquaintance. Your behavior then may be to initiate a conversation with the person based on the smile being an assumed symbol or sign of cordiality. 2

(B) As we try to guess the intentions of the other, the other is impacting our behavior. For example, if you smile at someone while at a party, it may be your way of attempting to make the person's acquaintance. 1

(C) We also assume that others we encounter will behave as we would behave if we were in their roles, that they will conform to a community standard or model. By adapting our behavior accordingly, the expected behavior or the perspective of the other is acting upon us. 0


① A-C-B ② B-A-C ③ B-C-A ④ C-A-B ⑤ C-B-A

 


160. 20수특 T1-13

 

Which comes first, the site or the plan for the house?


(A) I believe a house should "grow" from its site and not look like it has been imported and dropped there haphazardly. If you have already selected your architect, bring him along to offer his opinion on the sites you are considering. He may well see things about a site that you may not see. 1

(B) I advise my clients to choose the site first. This allows you to design a house to fit the land. You wouldn't buy a rug and then figure out what room it fits in later on. 0

(C) The site you have in mind might not be appropriate for the house you desire. How many times have we seen houses placed awkwardly on a slope when the house design would clearly have been more comfortable on a flat site? Developments of tract houses are fertile ground for examples of house designs that were poorly fitted to their sites. These mismatches of house to site are akin to wearing a tuxedo with tennis shoes. 2


① A-C-B ② B-A-C ③ B-C-A ④ C-A-B ⑤ C-B-A

 


161. 20수특 T1-14

 

Office workers can sometimes choose their own desk setups, integrating exercise on an individual basis.


(A) In the laboratory, regular exercise improves problem-solving abilities, fluid intelligence, and even memory — sometimes dramatically so. It's worth finding out whether the same is true in business settings, too. 2

(B) But businesses have compelling reasons to incorporate such radical ideas into company policy as well. Business leaders already know that if employees exercised regularly, it would reduce health-care costs. There's no question that halving someone's lifetime risk of a debilitating stroke or Alzheimer's disease is a wonderfully humanitarian thing to do. 0

(C) But exercise also could boost the collective brain power of an organization. Fit employees are more capable than sedentary employees of mobilizing their God-given IQs. For companies whose competitiveness rests on creative intellectual horsepower, such mobilization could mean a strategic advantage. 1


① A-C-B ② B-A-C ③ B-C-A ④ C-A-B ⑤ C-B-A

 


162. 20수특 T1-15

 

It is very important in the information age to understand the difference between knowledge and information.


(A) What is accessible by computer and, indeed, what is published in the journals is information. Knowledge is something that has to be constructed in the mind of the expert reader. This is what scholarship is about. 0

(B) It would probably take many years, despite the fact that their research was all published. To take another example, what do producers of science documentaries for television programmes do when they are researching their subjects? They talk to the experts rather than trying to read the journals. Quite rightly, as that is the only place that knowledge is to be found — inside the heads of the scholars. 2

(C) Information is, these days, instantly accessible, but knowledge still takes years of dedicated study to acquire. Imagine that a freak accident wiped out an entire field of experts on a subject while all were attending a conference. How long would it take to reconstruct expertise in the field so that research could once again progress? 1


① A-C-B ② B-A-C ③ B-C-A ④ C-A-B ⑤ C-B-A

 


163. 20수특 T1-16

 

When people want to react correctly to a persuasive message but don't have the motivation or ability to think about it deeply, there is a shortcut they can take.


(A) In addition, the more consensus you witnessed among audience members, the more likely you would be to follow their lead, even if you didn't initially agree with them. It's for this reason that interrogators are taught to say to a suspect "We believe you are guilty" rather than "I believe you are guilty." 2

(B) They can observe the responses of others to the message. 0

(C) For example, if under such conditions you heard a political speech and everyone in the audience around you responded enthusiastically to it, you might well conclude that the speech was a good one and become persuaded in its direction. 1


① A-C-B ② B-A-C ③ B-C-A ④ C-A-B ⑤ C-B-A

 


164. 20수특 T1-17

 

Popular participation in conflict in the nineteenth century cannot be understood without examining the role of empire.


(A) Scots, for example, were excluded from membership of the English community, but by participating in the wars against France and the empire in India, they became part of the British nation, increasing the war-fighting capacity of the latter. Empire was a crucial mechanism in constructing the exchange in the nineteenth century. 2

(B) Hannah Arendt later dismissively characterized imperialism as "the export of superfluous men and superfluous capital." She was correct that colonial functionaries were often from marginal populations — Cecil Rhodes, after all, would advocate imperialism "to settle surplus population" and thus "avoid civil war" — but these marginalized individuals increasingly used their participation in colonial conflicts to make claims to membership in the home nation. 1

(C) Individuals had always gone to the colonies for profit or to settle. But individuals also participated in nineteenth-century imperial campaigns for a novel reason. 0


① A-C-B ② B-A-C ③ B-C-A ④ C-A-B ⑤ C-B-A

 


165. 20수특 T1-18

 

Agricultural effects on water quality can occur at local, regional, and national scales.


(A) For example, increased levels of nutrients from agricultural fertilizers can stimulate algal blooms and affect the ecology of local streams. Nitrate and some chemical weed killers can move through the soil to groundwater and, eventually, to local streams. 0

(B) Elevated nutrient inputs stimulate harmful algal blooms along the US coasts causing negative economic impacts. 2

(C) Farther downstream, these elevated nutrients can increase costs associated with treating the water so that it is suitable for drinking. Ultimately, chemicals associated with agricultural activities (such as nutrients and pesticides) and sediment (eroded soil) empty into our river mouths and can harm valuable commercial and recreational fisheries. 1


① A-C-B ② B-A-C ③ B-C-A ④ C-A-B ⑤ C-B-A

 


166. 20수특 T1-19

 

Many sociologists identify themselves as researchers to the people they study.


(A) On the surface, the neighborhood looked like a badly organized place, an urban jungle of its period. Yet Gans discovered that it was a well-organized community ― an urban village rather than a jungle — where the residents enjoyed close social relationships with one another. 2

(B) They do not worry that revealing their true identity will change their subjects' behavior. They are not overly concerned that subjects will hide secrets from them. Usually, they strive to minimize these problems by not getting too deeply involved with their subjects while simultaneously establishing a good rapport with them. 0

(C) This is not easy to accomplish, though. Nevertheless, such efforts have paid off, as indicated by some sociological insights that have emerged from their work. Herbert Gans, for example, became a participant observer in a poor Italian neighborhood in Boston in the late 1950s. 1


① A-C-B ② B-A-C ③ B-C-A ④ C-A-B ⑤ C-B-A

 


167. 20수특 T1-20

 

Only 10 percent of the midlatitude rainforest in the Pacific Northwest remains untouched.


(A) Throughout the tropics, a pattern of replacing numerous species with a few favored ones is common. In particular eucalyptus has been preferred over existing local species because it is fast growing and, when cut for coppicing, its shoots quickly develop into new tree growth. However, the oil in its leaves results in a ground litter that inhibits undergrowth with the result that soil erosion occurs. Hence, reforestation utilizing eucalyptus trees is not environmentally friendly. 2

(B) Forests in their natural condition once masked the area. This has been replaced by a checkered mosaic composed of plots of bare ground, recently replanted commercial saplings, and some mature forests. This type of land cover threatens the habitat of many plants and animals. 0

(C) Current conditions represent a chronically degrading environmental condition. The U.S. Forest Service, under public pressure and court orders, has altered the former approved clear-cutting strategy. Now some mature trees are left standing with the hope of encouraging a more natural regeneration of forest lands. 1


① A-C-B ② B-A-C ③ B-C-A ④ C-A-B ⑤ C-B-A

 


168. 20수특 T1-21

 

One particular problem that people sometimes struggle with is avoiding the use of masculine pronouns to refer to both men and women.


(A) Students often argue that using "he" or "his" is less cumbersome than writing "he or she" or "his or her." As an alternative, you can rephrase a sentence to use a gender-neutral plural pronoun. 0

(B) Instead of saying "Man cannot live without water," you could say "One cannot live without water" or "People cannot live without water." Avoiding sexist language isn't all that difficult, but it may take conscious effort; the pay-off is more effective interpersonal communication. 2

(C) The sentence "A doctor should be polite to his patients" excludes the possibility of female doctors, but "Doctors should be polite to their patients" makes the same point without the gender bias. In the same way, you can replace masculine terms to refer to humans with gender-inclusive alternatives. 1


① A-C-B ② B-A-C ③ B-C-A ④ C-A-B ⑤ C-B-A

 


169. 20수특 T1-22

 

The need for distinctiveness is a basic human need to which sport fandom can contribute.


(A) Given that brand consumption is associated with desires to be unique, it seems reasonable that individuals use sport fandom as an opportunity to meet their need for distinction by selectively choosing to follow non-mainstream sports or less popular teams. For instance, individuals can partially meet their need for uniqueness by identifying with a distant team or rooting for an underdog. 2

(B) According to this framework, individuals strive for two sometimes opposing social goals: inclusion and differentiation. These goals are best satisfied "through identification with distinctive groups that satisfy both needs simultaneously." 1

(C) Although humans want to feel a sense of belonging with those around them, they simultaneously have a need to be different and unique. The fundamental need for uniqueness is best understood through Brewer's Optimal Distinctiveness Theory. 0


① A-C-B ② B-A-C ③ B-C-A ④ C-A-B ⑤ C-B-A

 


170. 20수특 T1-23

 

Chunking is vital for cognition of music.


(A) you don't reel off the names of roads as an abstract list, but have to construct your route by mentally retreading it. When musicians make a mistake during rehearsal, they wind back to the start of a musical phrase ('let's take it from the second verse') before restarting. 2

(B) If you ask a pianist to start a Mozart sonata from bar forty-one, she'll probably have to mentally replay the music from the start until reaching that bar —. the score is not simply laid out in her mind, to be read from any arbitrary point. It's rather like describing how you drive to work:. 1

(C) If we had to encode it in our brains note by note, we'd struggle to make sense of anything more complex than the simplest children's songs. Of course, most accomplished musicians can play compositions containing many thousands of notes entirely from memory, without a note out of place. But this seemingly awesome accomplishment of recall is made possible by remembering the musical process, not the individual notes as such. 0


① A-C-B ② B-A-C ③ B-C-A ④ C-A-B ⑤ C-B-A

 


171. 20수특 T1-2425

 

What is wrong with pseudoscience?


(A) Differences of opinion among its believers, when such disagreements arise at all, lead to endless sect splitting rather than to progress. Fourth, the primary aim of pseudoscience is not to set up, test, and correct systems of hypotheses (theories) mapping reality, but to influence things and men:. it has, like magic and like technology, a primarily practical aim rather than a cognitive one but, unlike magic, it presents itself as science and, unlike technology, it does not enjoy the backing of science. 2

(B) the pseudoscientist, like the fisherman, exaggerates his catch and neglects his failures or excuses them. Third, pseudoscience lacks a self-correction mechanism:. it cannot learn from either fresh empirical information (which it swallows without digesting it), new scientific discoveries (which it looks down on), or criticism (which it rejects furiously). It can make no progress because it manages to interpret every failure as confirmation and every criticism as an attack. 1

(C) Certainly not just that it is basically false, since anyway all our factual theories are at best partially true. What is wrong with pseudoscience is, first, that it refuses to ground its doctrines and could not do it because pseudoscience makes a total break with our scientific heritage - which is not the case of scientific revolutions, all of which are partial since every new idea has to be judged by means of others that are not questioned in the given context. Second, pseudoscience refuses to test its doctrines by experiment proper;. moreover, it is largely untestable because it tends to interpret all data in such a way that its theses are confirmed no matter what happens:. 0


① A-C-B ② B-A-C ③ B-C-A ④ C-A-B ⑤ C-B-A

 


172. 20수특 T1-2628

 

On a beautiful spring afternoon, a softball game was held between rivals Central Washington University and Western Oregon University.


(A) The only home run of Tucholsky's four-year career would be erased. The coach didn't know what to do. That's when Mallory Holtman stepped in. Holtman played first base for the other team. She was also Central Washington's all-time home run leader. She knew that if her team lost the game, their playoff hopes would probably be gone. But after listening to the coach and umpire for a while, she asked a question. Is the other team allowed to carry her around the bases? The umpires said there was nothing in the rule book against it. So Holtman and Central's shortstop, Liz Wallace, walked over and helped Tucholsky up. Carrying her, they resumed the home run walk. At each base, they paused to let Tucholsky touch it with her uninjured leg. 1

(B) Holtman said, "I wonder what this must look like to other people." The three players burst out laughing. The other people in the stadium weren't laughing, though. They were shedding tears at seeing such a moving act of sportsmanship. Tucholsky's Western team ended up winning the game 4 to 2. Afterward, someone realized that the umpires had made a mistake. The rules did allow a substitute runner, after all. But it didn't really matter. "In the end, it's not about winning and losing so much," said Holtman. "It was about this girl. She hit it over the fence and was in pain. And she deserved a home run." 2

(C) Western's Sara Tucholsky had never hit a home run - not even in batting practice. Then, in the second inning she smashed the ball over the center field fence. With two players on base, it would be a three-run shot. Thrilled, Tucholsky sprinted toward first base. But as she watched the ball clear the fence, she missed first base. When she stopped quickly to go back and touch it, something in her knee gave out. She fell to the ground. In terrible pain, Tucholsky crawled through the dirt back to first base. The Western coach rushed onto the field. The umpires told the coach it was against the rules for Tucholsky's Western teammates to help her around the bases. The coach could substitute a runner for her. But then the hit would be judged a single. 0


① A-C-B ② B-A-C ③ B-C-A ④ C-A-B ⑤ C-B-A

 


173. 20수특 T2-1

 

Our school library is very nice, and I am proud of being a member of the library maintenance team.


(A) And as a student and a member of the team, I would like to suggest a few improvements that would make our library even better. First, a library should be a quiet place, but noise from the school cafeteria can easily be heard in the school library. 0

(B) Finally, a magazine and newspaper section should be created so that students can come to the library to read about current events. Thank you in advance for considering my suggestions. I look forward to talking with you about them. 2

(C) Therefore, I think the library should be moved to the first floor so that the noise can be minimized. There is also a need for additional bookshelves to accommodate the books that have been piled up in a corner of the library. 1


① A-C-B ② B-A-C ③ B-C-A ④ C-A-B ⑤ C-B-A

 


174. 20수특 T2-2

 

The day of the salsa contest arrived and practically all the local dance schools took part.


(A) The hostess introduced Annette and Reiner, from Diego Santiago's dance school, on stage. They walked up onto the dance floor. While they bowed to the jury and the audience, Reiner noticed that Annette was trembling with anxiety. 0

(B) He squeezed her hand and Annette felt the warmth that radiated from his hand flowing like calming energy through her body. Her goosebumps disappeared and were replaced by excitement and joyful anticipation. Annette stood in the spotlight in front of a large crowd and felt not afraid but absolutely supported by her partner. It was as though, when he held her hand, he absorbed all her fears, leaving her with only positive feelings. 2

(C) "Don't worry," he encouraged her in a low voice. "You were born to dance. We'll be OK!" 1


① A-C-B ② B-A-C ③ B-C-A ④ C-A-B ⑤ C-B-A

 


175. 20수특 T2-3

 

A big mistake many new college students make is just sitting in their rooms.


(A) Walk down your residence hall corridor and look into other rooms where doors are propped open. If you see a student sitting alone in a room, knock lightly, say hello, and introduce yourself. If nothing else, get out and walk around campus. 1

(B) Some students stay in their rooms waiting for friends to come find them, convinced that it will happen on its own. Other students remain in their rooms because they feel shy or are uncertain about how to approach other people. For the first few weeks of the semester, you should try to spend as little time as possible in your room. 0

(C) Look for groups of new students who are hanging out together and join them. Whatever you do, don't isolate yourself. Most students form friendships within the first couple of months of college, and it is much harder to join existing groups than it is to meet new people one-on-one. 2


① A-C-B ② B-A-C ③ B-C-A ④ C-A-B ⑤ C-B-A

 


176. 20수특 T2-4

 

The freedom to choose one's identity is critical, since the sources of identity are shifting from "belonging" to "achievement."


(A) A good society recognizes and does not pit roots and wings against one another. If roots are the necessary condition for happiness, then wings are the sufficient condition. A good life is not possible without both. 2

(B) Speaking for many, Kymlicka says that "identification is more secure, less liable to be threatened, if it does not depend on accomplishment." But this is absurd. Achievement increasingly is the basis for a satisfying life. To be sure, some people want unconditional acceptance by their "in" group. 0

(C) But more and more, people in rich countries achieve many of their identities. They choose their careers, friendships, allies, mixing and matching pieces and styles. Even their ethnic, racial and national affiliations are forged in various ways, despite the fact that a person's self-image depends partly on how he's viewed by others. Indeed, the ability to make one's own self is the essence of freedom. 1


① A-C-B ② B-A-C ③ B-C-A ④ C-A-B ⑤ C-B-A

 


177. 20수특 T2-5

 

The efficient and valuable use of big data needs the personal and organizational capacity of asking the right questions and in the right way.


(A) The arts/humanities are important in the age of digital transformation and big data because they dominate the knowledge domains of the creation and communication of narratives as well as meanings of human life. In other words, the arts and humanities are capable of embedding into big data the aesthetic human-based dimensions that ultimately make them relevant in order to identify, address, and solve key questions for sustainable societal, economic, and environmental wealth creation. 1

(B) Big data is powerful only if is generated, combined, or supported by the creation of strong narratives, organizationally and contextually framed. This means that the big data has to be "thick," i.e., not only quantitatively but most importantly qualitatively relevant. 0

(C) The arts and humanities are essential in order to make big data, analytics, data mining, and digital transformation significant for stakeholders. 2


① A-C-B ② B-A-C ③ B-C-A ④ C-A-B ⑤ C-B-A

 


178. 20수특 T2-6

 

Once you have firmly established the habit of placing a pair of commas around a nonessential element that interrupts or changes the normal order of the English sentence, you can consider a few situations in which this mark of punctuation may safely be omitted.


(A) You are doubtless aware that the tendency of modern writers is to make considerably less use of punctuation than their predecessors did. 0

(B) These marks were never marks of actual punctuation, in the sense that they clarified meaning, and today they have almost disappeared from printing. A more important reason for the diminishing amount of punctuation in modern writing is that our writers are learning to construct their sentences in such a way that the word-symbols themselves communicate the meaning clearly. 2

(C) One reason for this, of course, is that we have ceased to use the "musical notation" that was once fashionable, probably because most modern prose is designed to be read silently, to be taken in by the eye and not by the ear. 1


① A-C-B ② B-A-C ③ B-C-A ④ C-A-B ⑤ C-B-A

 


179. 20수특 T2-7

 

Like the downtown office complex, tourism has frequently developed as islands of renewal in seas of decay.


(A) For tourists, the city can be reduced to a simulacrum, a set piece representing the city in its entirety. Thus, reduced to Harborplace or the Renaissance Center and Greektown, both Baltimore and Detroit can be presented as gleaming new places to play. 2

(B) In a hostile environment, zones of demarcation can solve seemingly insolvable problems of image and social control. Tourists who visit converted cities are unlikely to see the city of decline at all, except on their way from an airport. 1

(C) The strategy of carving out sharply demarcated and defended zones for middle-class consumers of entertainment and leisure came naturally to older cities confronted with problems of crime, poverty, and physical neglect. Creating a "tourist bubble" was tempting ― some might say necessary ― as a way not only of securing a space for development, but for achieving an efficient application of scarce resources. 0


① A-C-B ② B-A-C ③ B-C-A ④ C-A-B ⑤ C-B-A

 


180. 20수특 T2-8

 

Henry Cavendish was born in 1731 in Nice, where his parents had gone because of his mother's health, which continued to fail.


(A) From there, he proceeded to St. Peter's College (Peterhouse), Cambridge University in 1749, leaving after three years without a degree. For the next thirty-odd years, he lived at his father's house on Great Marlborough St., London. Freed from the need to support himself, he followed his inclinations, which were to study and carry out researches in the physical sciences. 1

(B) Around the time his father died, in 1783, he acquired two houses of his own, one in and one outside London, both of which he adapted to his scientific habit. He was a prominent member and administrator of the Royal Society of London. His manner of living was modest, and over time he accumulated an immense fortune. He died in 1810, at age 78. 2

(C) She died two years later, after giving birth to a second son. Henry's father never remarried. When Henry was 11, he was enrolled in Hackney Academy, a progressive school outside London. 0


① A-C-B ② B-A-C ③ B-C-A ④ C-A-B ⑤ C-B-A

 


181. 20수특 T2-9

 

The graph above shows U.S. adults' evaluation of the health risks of food additives, sorted by gender.


(A) The lowest percentage of both men and women say food and drinks with artificial coloring pose a serious health risk. 2

(B) Women (39%) are more wary than men (25%) of meat from animals given antibiotics or hormones. Men and women show the smallest difference in their evaluation of serious health risks when it comes to food and drink with artificial preservatives. 1

(C) Women are more inclined than men to view food additives as a serious health risk: More women (55%) than men (46%) say that additives in the food people eat every day pose a serious risk to their health. About four in ten women say fruits and vegetables grown with pesticides pose a great deal of health risk, while about one in four men say the same. 0


① A-C-B ② B-A-C ③ B-C-A ④ C-A-B ⑤ C-B-A

 


182. 20수특 T2-10

 

2020 Jacksonville's Annual Aisle of Lights One night a year, Jacksonville shines with thousands of twinkling lights for the Aisle of Lights celebration.


(A) The luminaria kit includes candles, paper bags, and sand. 3. Arrange the bags, each with a candle inside and weighted with sand in the bottom, along your sidewalk. 4. Light the candles on Sunday, December 13, around 6 pm and enjoy! 1

(B) 5. Don't worry about cleaning up the luminarias. Volunteers will collect them the next morning. Other Winter Activities: In addition to the Aisle of Lights celebration, there will be other festive activities around town throughout the weekend of December 12 and 13. See a listing of the activities at www.jacksonville.org/WinterActivities. 2

(C) Since 1990, on every second Sunday of December, Jacksonville residents and businesses have celebrated this annual community tradition. And on December 13 this year, every sidewalk of Jacksonville will shine again. How to Participate 1. Visit the Jacksonville Community Center from December 6 to December 12 to fill out a brief registration form and get a free luminaria kit. 2. 0


① A-C-B ② B-A-C ③ B-C-A ④ C-A-B ⑤ C-B-A

 


183. 20수특 T2-11

 

City of London Public Bike Rental Service 1.


(A) 3. RETURN YOUR BIKE You can return your bike to any station. To return your bike, find your chosen bike station and push the front wheel of your bike into an empty spot. If you have inserted the bike correctly, a green light will come on above the keypad next to the front wheel. If your bike is not returned within 24 hours, or if you have damaged the bike, you may be subject to a fine. 2

(B) OBTAIN YOUR BIKE Choose a bike and enter your printed code into the keypad. When the green light appears, pull your bike free. Adjust the seat height. Now you're ready to ride! 1

(C) HIRE YOUR BIKE At your chosen bike station, first touch the screen and follow the basic instructions for payment. Have your debit or credit card ready, as this is the only accepted method of payment. Upon payment, you will be given a printed code to unlock your bike. 2. 0


① A-C-B ② B-A-C ③ B-C-A ④ C-A-B ⑤ C-B-A

 


184. 20수특 T2-12

 

Vagueness is an obstacle to efficient communication.


(A) Sometimes people who want to avoid committing themselves to a particular course of action use vagueness as a ploy. 0

(B) A good journalist would then press for further information about precisely how this efficiency was to be achieved, forcing him to come out from behind this veil of vagueness. Or someone who was late for an appointment but didn't want to admit that this was because he'd stopped for a drink on the way might say 'Sorry I'm late, I had something I needed to do on the way here and it took slightly longer than I expected', deliberately leaving the cause of the delay vague, and exercising a particular kind of economy with the truth. 2

(C) For instance, a politician asked how precisely he intends to save money in the public sector might make vague generalisations about the need for improved efficiency, which, while true, don't commit him to any particular way of achieving this. 1


① A-C-B ② B-A-C ③ B-C-A ④ C-A-B ⑤ C-B-A

 


185. 20수특 T2-13

 

When opposites blend, they are placed on the far ends of a continuum, and between the two extremes there is a gradation that mixes the two opposites.


(A) For instance, black and white blend into each other through shades of gray. As the amount of white decreases through shades of gray, the amount of black increases. 0

(B) In a blend, pure black and pure white are diluted when combined into gray. They both lose their identity; gray is not black and it is not white. 2

(C) The two opposites are always in a proportional relationship, but at any point along the continuum there is some amount of each (except at the very extremes). In contrast, a zero-sum game where the winner takes all also establishes a proportional relationship, but at any point along the continuum there is only one or the other, black or white, and each retains its full identity.1


① A-C-B ② B-A-C ③ B-C-A ④ C-A-B ⑤ C-B-A

 


186. 20수특 T2-14

 

In sports, attendance is nearly always (98-99 percent of the time) with at least one other person.


(A) The excitement of the competition and the aura of the star power of the players on the team are such that the experience is best enjoyed in the presence of others. 2

(B) The sports fan pays a price for the right to enjoy an emotional experience with others. The fan goes to the game to be with others, to share the experience in this social exchange. 0

(C) More broadly speaking, unlike most other retail settings, large crowds have positive psychological effects. No line at the grocery checkout will make most shoppers happy, but no line to see a ballgame is a definite hint to a fan either that this is a terrible sporting event or that the fan has arrived at the stadium on the wrong date. 1


① A-C-B ② B-A-C ③ B-C-A ④ C-A-B ⑤ C-B-A

 


187. 20수특 T2-15

 

As writers, we need to share our experiences of writing and to encourage one another, but we also need to work independently.


(A) Writing needs that stocked reservoir of real but aimless sensations. When the time comes and you need them, they'll find you. 2

(B) So the point you might consider is this: if you have a piece of writing in mind, or in some half-finished state — which is actually the case for almost all writers much of the time — stop, go for a walk, draw a picture, enjoy something different, go window-shopping, or even fly-fishing, but observe, take notice, bring to your surroundings and other people an extra special level of attention, see them as unfamiliar and worth all your attention, store up what you see, hear or think, and let it lie in that part of your mind which has no obvious practical function, which isn't continually worrying about what you have to do today or next week. 1

(C) We may also need to do certain things which, paradoxically, do not involve words. 0


① A-C-B ② B-A-C ③ B-C-A ④ C-A-B ⑤ C-B-A

 


188. 20수특 T2-16

 

When asked, "what was one of your best days at work?"


(A) It was not because of the hardship, per se, but because the hardship was shared. It is not the work we remember with fondness, but the fellowship, how the group came together to get things done. 2

(B) For most of us, we have warmer feelings for the projects we worked on where everything seemed to go wrong. We remember how the group stayed at work until 3 a.m., ate cold pizza and barely made the deadline. Those are the experiences we remember as some of our best days at work. 1

(C) very few of us recount the time everything went smoothly and the big project we were working on came in on time and under budget. Considering how we work so hard to make things go well, that example should count as a pretty good day at work. But strangely, the days everything goes smoothly and as planned are not the ones we remember with fondness. 0


① A-C-B ② B-A-C ③ B-C-A ④ C-A-B ⑤ C-B-A

 


189. 20수특 T2-17

 

If you're a small herbivore, it's a bad idea to run in the face of danger.


(A) And because herds are easier to spot than single animals, roe deer live alone. But another reason for their solitary existence is the lack of food in ancient undisturbed forests. A herd of deer would have to cover a lot of territory to find sufficient food. 1

(B) Travelling long distances, however, increases the risk of coming across a pack of wolves. And so the single life is better. 2

(C) Wolves could easily follow you and attack, so it's better for you to hide. Roe deer don't run very far before they tum around and try to return to their original location, and when they do, they cross their own tracks, which confuses their pursuers ― which trail should they follow? Once they're safely back on home grass, roe deer hide in groups of small trees. 0


① A-C-B ② B-A-C ③ B-C-A ④ C-A-B ⑤ C-B-A

 


190. 20수특 T2-18

 

It was not until the advent of the Industrial Revolution — the historical period beginning in the late 1700s when the economies of the United States and many nations in Europe shifted from manual labor and hand tools to machines and factory manufacturing ― that human-produced garbage became a critical issue.


(A) As people were able to buy more and more new things, they stopped seeing value in broken and used items and began seeing these old items as trash. New forms of colorful advertising and packaging encouraged this burgeoning consumer culture. 2

(B) This economic change produced many more products for people to purchase, and it also created jobs that helped to increase wealth that could be used to purchase products. 1

(C) The full effect of the Industrial Revolution, however, was not felt until around the turn of the twentieth century, when systems of mass production and mass distribution were developed. 0


① A-C-B ② B-A-C ③ B-C-A ④ C-A-B ⑤ C-B-A

 


191. 20수특 T2-19

 

The reasons for the deficiencies in human-machine interaction are numerous.


(A) But in fact, we humans are amazingly complex. Those who have not studied human behavior often think it is pretty simple. Engineers, moreover, make the mistake of thinking that logical explanation is sufficient: "If only people would read the instructions," they say, "everything would be all right." 2

(B) Why this deficiency? Because much of the design is done by engineers who are experts in technology but limited in their understanding of people. "We are people ourselves," they think, "so we understand people." 1

(C) Some come from the limitations of today's technology. Some come from self-imposed restrictions by the designers, often to hold down cost. But most of the problems come from a complete lack of understanding of the design principles necessary for effective human-machine interaction. 0


① A-C-B ② B-A-C ③ B-C-A ④ C-A-B ⑤ C-B-A

 


192. 20수특 T2-20

 

When you buy from large corporations, you support the increasing consolidation of wealth and power in the hands of the few.


(A) Paying in cash, rather than by credit card, can also help local businesses as they are often the ones least able to afford the hefty fees the credit card companies charge them for each and every transaction. Look in the phone book for local alternatives to large corporate chains. 2

(B) Chain businesses often take those dollars directly away from smaller local businesses that cannot afford to lose the income. By making your purchases at local businesses, you spread that wealth out to more local people and increase your community's standard of living. 0

(C) This is because local businesses rely more on local suppliers and service providers, forming a kind of local economic web of interdependence that creates jobs and a thriving community. Therefore, every dollar you spend at a local business helps your community maintain its individual character, uniqueness, and diversity while supporting your neighbors in their quest for the good life. 1


① A-C-B ② B-A-C ③ B-C-A ④ C-A-B ⑤ C-B-A

 


193. 20수특 T2-21

 

Nitrogen in its gaseous form is often used in situations in which it is important to keep other, more reactive atmospheric gases away.


(A) In addition to these applications, nitrogen is used in oil production, in which it is pumped in compressed form underground to force oil to the surface. Ordinary air cannot be used for this purpose because some of the gases that make up air would react with the oil, producing undesired by-products. 2

(B) Nitrogen has recently also been used in blanketing fruit after it has been picked to protect it from rotting. Apples, for example, can be stored for up to 30 months if they are kept at low temperatures in an atmosphere of nitrogen. 1

(C) It serves industry as a blanketing gas, for example, in protecting materials such as electronic components during production or storage. To prevent the oxidation of wine, wine bottles are often filled with nitrogen after the cork is removed. 0


① A-C-B ② B-A-C ③ B-C-A ④ C-A-B ⑤ C-B-A

 


194. 20수특 T2-22

 

The bigger the group, the greater the benefits — up to a point.


(A) Even with the benefit of gossip to circulate information about other people's reputations, to boost our social capacities, if a group of humans has more than 150 members, we end up losing track of who's who. That makes the maintenance of social harmony within the group much more challenging. For human cooperatives to remain stable across populations larger than 150 people, we needed to invent God (or gods). 2

(B) It is thought to reflect limitations in how much social information a human brain can keep track of, not just regarding their own relationships, but other people's too. Our capacity to sustain larger cooperative groups than any other primate probably stems from our ability to learn not just from our own personal experience, but also from other people's experiences. 1

(C) Communities of humans tend to be relatively stable up to around 150 people. This appears to be the optimal size for a cooperative group of humans both across the world and throughout history. 0


① A-C-B ② B-A-C ③ B-C-A ④ C-A-B ⑤ C-B-A

 


195. 20수특 T2-23

 

How can we make decisions in the face of scientific uncertainty?


(A) An approach currently favored by many natural resource managers is called adaptive management, or "learning by doing." In adaptive management, policies are designed from the outset to use scientific principles to examine alternatives and assess outcomes. 1

(B) The answer is that our plans generally have to be conditional and contextual. Scientific information can help us understand environmental issues, but the policies we create based on this understanding will always depend on further study and more confirming evidence. 0

(C) Rather than assume that what seems the best initial policy option will always remain so, adaptive management sets up scientific experiments to monitor how conditions are changing, and what effects our actions (or inactions) are having on both target and nontarget elements of the system. The goal of adaptive management is to enable us to live with the unexpected. It aims to yield understanding as much as to produce answers or solutions. 2


① A-C-B ② B-A-C ③ B-C-A ④ C-A-B ⑤ C-B-A

 


196. 20수특 T2-2425

 

Recent research on solutions to social dilemmas provides an example of the positive value of regulatory authorities.


(A) In a social dilemma, a society must prevent citizens from engaging in actions that are individually beneficial in the short term but that hurt society in the long term. Studies suggest that one solution that groups voluntarily adopt when faced with social dilemmas is to designate formal leaders who are empowered to control the behavior of the group's members. Similarly, groups develop rules governing members ' conduct to preserve valuable social relationships. 0

(B) It cannot be assumed that authorities will be compassionately motivated and will use their power and legitimacy to promote the positive objectives outlined above. Although they can facilitate the productive exchange of resources to the benefit of all members of society, it is not inherent in the nature of authority that it will function in this way. The effects of authority depend on the motives of those exercising it. 2

(C) These informal rules are the precursors of formalized law. It is also important to recognize the potential dangers of giving authorities the power to affect public behavior. Authorities may use that power to advance limit their own interest, or the interest of a particular group or individual, over the interest of others. 1


① A-C-B ② B-A-C ③ B-C-A ④ C-A-B ⑤ C-B-A

 


197. 20수특 T2-2628

 

There was a young boy who used to go for regular soccer practice but always played in reserves and never made it to the playing 11.


(A) He said, "OK, son, go, play. But remember, I am going against my better judgment and the reputation of the school is at stake. Don't let me down." The game started and the boy played like a house on fire. Every time he got the ball, he scored a goal. Needless to say, he was the best player and the star of the game. His team had a spectacular win. When the game finished, the coach went up to him and said, "Son, how could I have been so wrong in my life? I have never seen you play like this before. 1

(B) While he was practicing, his father used to sit at the far end, waiting for him. The matches had started and for four days, he didn't show up for practice or quarter or semifinals. All of a sudden he showed up for the finals, went to the coach and said, "Coach, you have always kept me in the reserves and never let me play in the finals. But today, please let me play." The coach said, "Son, I'm sorry, I can't let you. There are better players than you and besides, it is the finals, the reputation of the school is at stake and I cannot take a chance." The boy pleaded, "Coach, I promise I will not let you down. I beg of you, please let me play." The coach had never seen him plead like this before. 0

(C) What happened? How did you play so well?" The boy replied, "Coach, my father is watching me today." The coach turned around and looked at the place where the boy's father used to sit. There was no one there. He said, "Son, your father used to sit there when you came for practice, but I don't see anyone there today." The boy replied, "Coach, there is something I never told you. My father was blind. Just four days ago, he died. Today for the first time he is seeing me (from above)." 2


① A-C-B ② B-A-C ③ B-C-A ④ C-A-B ⑤ C-B-A

 


198. 20수특 T3-1

 

It has come to my attention that you were responsible for this past weekend's "Halloween Scare" email that was circulated through our corporation's intranet mail system.


(A) Thank you in advance for your cooperation. 2

(B) Your expertise as a systems analyst is vital to us, but your outstanding ability does not justify your singular poor judgment. In the future, please limit your use of The Dogwood Firm's technology to work-specific business. 1

(C) The rather large graphic file you created and mass-mailed to all departments and all personnel was not only thematically and visually inappropriate, but it overloaded our servers and crashed key hubs in Charlotte, Biloxi, and Mobile, temporarily crippling business in the Southern Region. You must know that this kind of behavior cannot be condoned at The Dogwood Firm. 0


① A-C-B ② B-A-C ③ B-C-A ④ C-A-B ⑤ C-B-A

 


199. 20수특 T3-2

 

One evening, just three days before the Olympics were to begin, my father called me into the kitchen.


(A) I guess I'll just have to get a job since I didn't get into any of the schools I wanted to go to." "Well," he said, "you might not want to start filling out any job applications too soon." When I gave him a confused look, he smiled broadly. 1

(B) With a very stern look, he said, "Lisa, what do you plan to do after the Olympics are over?" His question was like a bucket of ice-cold water being poured on me. Forced to face the reality beyond the summer, I answered rather weakly, "I don't know, Dad. 0

(C) "Lisa, after you go to the Olympics — you're going to college!" Then he handed me a letter that stated that I had been accepted and was receiving financial grants to go to a four-year university! I screamed and jumped up and down. My father gave me a huge hug and we danced around the kitchen together, singing joyfully, "Go, Li-sa, Go, Li-sa" 2


① A-C-B ② B-A-C ③ B-C-A ④ C-A-B ⑤ C-B-A

 


200. 20수특 T3-3

 

Globalization and technological innovation are key drivers of socio-economic transformations.


(A) An important point is that globalization and technological innovation are not natural processes that societies must either endure or stop. 1

(B) Quite to the contrary, the particular ways in which globalization and innovation unfold can be shaped by policies and it is important to steer them in the direction of social inclusion. Therefore, not only should we make sure to support those who lose from the globalized economy and technological disruptions and ease their adaptation and transition to the new opportunities offered by these developments, but we can work to make the changes themselves occur in a way that generates less loss and more gain for all. 2

(C) Experts (not always decision-makers, unfortunately) know the virtues and dangers of the former, but there is much uncertainty about how the latter will affect quality of life and social inequalities. 0


① A-C-B ② B-A-C ③ B-C-A ④ C-A-B ⑤ C-B-A

 


201. 20수특 T3-4

 

Firms in the same industry may be able to imitate and copy one another more readily if they are located together.


(A) However, managers may not know which firm will develop leading innovations. On average, the "sharing" of information may benefit the group. 1

(B) Therefore, they may be able to respond to changes in their industry more quickly than if they were isolated from their competitors. Of course, the firm that is copied may be harmed, so in this instance, it would be better off in an isolated location, where copying would be more difficult. 0

(C) In industries with numerous and scattered innovations, such as fashion or computer games, all firms may be better off if they have locations that allow them to imitate quickly. Furthermore, a firm that copies two changes is in a better position to innovate additional changes by combining or modifying changes that were taken from other firms. Thus, particularly in fast-changing industries, economies from industrial imitation, modification, and innovation tend to be important sources of localization economies. 2


① A-C-B ② B-A-C ③ B-C-A ④ C-A-B ⑤ C-B-A

 


202. 20수특 T3-5

 

Workers are united by laughing at shared events, even ones that may initially spark anger or conflict.


(A) One team told repeated stories about a dumpster fire, something that does not seem funny on its face, but the reactions of workers motivated to preserve safety sparked laughter as the stories were shared multiple times by multiple parties in the workplace. Shared events that cause laughter can indicate a sense of belonging since "you had to be there" to see the humor in them, and non-members were not and do not. 1

(B) Instances of humor serve to enact bonds among organization members. Understanding the humor may even be required as an informal badge of membership in the organization. 2

(C) Humor reframes potentially divisive events into merely "laughable" ones which are put in perspective as subservient to unifying values held by organization members. Repeatedly recounting humorous incidents reinforces unity based on key organizational values. 0


① A-C-B ② B-A-C ③ B-C-A ④ C-A-B ⑤ C-B-A

 


203. 20수특 T3-6

 

As Marshall McLuhan famously said in the 1960s, we are now a post-literate society.


(A) Secondary schools and universities are trying desperately to keep these activities at the centre of schooling, but even some of the most conservative Ivy League schools have replaced listening to lectures and writing essays with field work, role-playing games, online discussion, and other forms of experiential, interactive "e-learning." Having a book published was once seen as the ultimate way of getting your message out to a wide audience. 1

(B) Today, a blogger can get millions of hits a day or an online video can have millions of views. If a book sells five thousand copies in Canada it is considered a bestseller. The Kony 2012 video now stands at over 96 million views — a shocking statistic for a thirty-minute video! 2

(C) Our culture has returned to a kind of medieval attitude toward print. Extended reading and writing is something small elite groups do. 0


① A-C-B ② B-A-C ③ B-C-A ④ C-A-B ⑤ C-B-A

 


204. 20수특 T3-7

 

The table above shows the American public's thoughts on which issues the president and Congress should prioritize in January 2011 and January 2019.


(A) The percentage of the public who considered improving the job situation a top priority fell by 34 percentage points over the eight-year period. Similarly, the percentage of the public who considered strengthening the nation's economy and reducing the budget deficit a top priority also declined, yet not as much as improving the job situation. 0

(B) The percentage of the public who said the president and Congress should prioritize the issues of improving education and securing Social Security did not change much during the period between 2011 and 2019. 2

(C) The percentage of the public who felt dealing with climate change should be prioritized increased substantially 一 18 percentage points 一 during the eight-year period. Additionally, reducing health care costs, dealing with problems of poor and needy people, and protecting the environment were the issues that had a higher percentage of support as top priorities in 2019 than in 2011. 1


① A-C-B ② B-A-C ③ B-C-A ④ C-A-B ⑤ C-B-A

 


205. 20수특 T3-10

 

An Ashanti funeral is often held several weeks after a death.


(A) They notify distant relatives, clean and tidy up the deceased person's house, and memorialize the departed family member on posters, T-shirts, and in other images that will be distributed to mourners. When all is ready, the body is brought back to the house and prepared for the funeral, covered with kente cloth and decorated with gold jewelry. 1

(B) During this period, while the body is preserved in a mortuary, the family of the deceased works to prepare for the event. Funerals are traditionally the responsibility of a person's abusua (matriclan); these are relatives who are linked through common maternal ancestors. 0

(C) The ceremonies can last several days. They begin with a lying-in-state at the family home, which culminates in the presentation of burial gifts for use by the deceased in his or her journey to asamando, the land of the dead. Then come the burial and finally the great funeral, a public event honoring the deceased person with music, dancing, food, and drinks. 2


① A-C-B ② B-A-C ③ B-C-A ④ C-A-B ⑤ C-B-A

 


206. 20수특 T3-11

 

Some scientists compare the brain to a relay station that merely coordinates incoming signals and outgoing responses, whereas others see it as an immense computer that processes information and then arrives at an appropriate response.


(A) When the mind selects and orders incoming information into meaning, it is telling itself a story. 2

(B) Francois Jacob, the French molecular biologist and Nobel laureate, suggests that the human mind is far more; it has a built-in need to create order out of the constant flow of information coming from its sensory organs. In other words, the brain creates a narrative, with a beginning, a middle and an end — a temporal sequence that makes sense of events. 0

(C) The brain selects and discards information to be used in the narrative, constructing connections and relationships that create a web of meaning. In this way, a narrative reveals more than just what happened; it explains why. 1


① A-C-B ② B-A-C ③ B-C-A ④ C-A-B ⑤ C-B-A

 


207. 20수특 T3-12

 

In general, searching online for health information can be valid, eye-opening, educational, and even useful.


(A) If you think that juice cleanses are the way to better health and well-being, it's easy to find websites supporting this. If delaying vaccines is your cup of tea, online sources abound. If you're debating whether to eat only organic food, plenty of available information will support this. 2

(B) While many doctors roll their eyes when they hear, "I did my research," from a patient, sometimes that research can be sound. If a patient has a rare disease and presents articles about it, many of us will be grateful that we were saved some extra work. But the Web becomes entangled when sites angled with opinions, personal anecdotes, exaggeration, and false claims manipulate the navigator to believe what is posted. 0

(C) People also run into trouble when looking for information online based on preconceived notions. Here comes the Curse of the Original Belief. If you believe that megadosing on vitamin C will prevent colds, you will seek out (and easily find) sites promoting this notion. 1


① A-C-B ② B-A-C ③ B-C-A ④ C-A-B ⑤ C-B-A

 


208. 20수특 T3-13

 

Beethoven himself raised the cash to visit Vienna at the age of 16 in the hope of persuading Mozart to give him lessons.


(A) If Beethoven had been able to prolong his stay in Vienna, he would surely have met Mozart in the end. But a summons home to his mother's deathbed prevented his persistence from being put to the test. 2

(B) Yet Beethoven may well have been right. Mozart, for all his youthfulness, already had such crookedly arthritic fingers that observers remarked on his inability to cut his meat. 1

(C) Of an actual meeting with Mozart, there is no evidence outside the fields of romantic biography. But he did, or so it seems, hear Mozart play, and complained of the 'choppiness' of his style ― a description which has to be matched against Mozart's own statement that his piano music was meant to 'flow like oil'. 0


① A-C-B ② B-A-C ③ B-C-A ④ C-A-B ⑤ C-B-A

 


209. 20수특 T3-14

 

It appears that Internet customers rarely secure the lowest price.


(A) Similarly, buyers normally use one online grocer because of the trouble of getting to know another site. In short, habits take over. 2

(B) According to a popular price comparison website, 80 percent of Internet customers pay more than they have to. It seems that use of the Internet to obtain better value is restrained by loyalty to particular websites. 0

(C) Once they are familiar with a site, consumers may return to it later because it is easy to use and saves time. A consumer might agree that a book might be cheaper elsewhere but still use the online bookstore they're familiar with because of convenience — this convenience can be seen as considerable, as often customers allow trusted sites to store their credit card and delivery details, so purchasing really is a single click. 1


① A-C-B ② B-A-C ③ B-C-A ④ C-A-B ⑤ C-B-A

 


210. 20수특 T3-15

 

Findings from several studies on nonpatient groups such as university students suggest that simply looking at everyday nature, as compared to built scenes that lack nature, is significantly more effective in promoting restoration from stress.


(A) One early study focused on students who were experiencing mild stress because of a final course exam. 0

(B) Results suggested that the nature views fostered greater psychological restoration as indicated by larger reductions in negative feelings such as fear and anger/aggression and much higher levels of positive feelings. Also, the scenes with vegetation sustained interest and attention more effectively than did the urban scenes without nature. 2

(C) A self-ratings questionnaire was used to assess restorative influences of viewing either a diverse slide sample of unblighted built settings lacking nature, or slides of undistinguished nature settings dominated by green vegetation. 1


① A-C-B ② B-A-C ③ B-C-A ④ C-A-B ⑤ C-B-A

 


211. 20수특 T3-16

 

Leonardo da Vinci had a keen interest in the reality and the wonders of nature as a broad and dynamic whole.


(A) This can be readily seen both in his drawings of anatomical structures in biology and his refined representations of mechanical structures in physics. He published amazingly detailed drawings of human anatomy, where, as one biographer noted, he paid "attention to the forms of even very small organs and hidden parts of the skeleton." 1

(B) Da Vinci is even credited with being the first in the modern world to introduce the idea of controlled experimentation 一 the core concept of science — and, for this, he has been considered by some writers to be the Father of Science. Probably more than any other scholastic luminary of that time, he recognized the relationship between the whole and its parts. 2

(C) The subject matter of his inspired paintings was almost more wondrous than reality. Da Vinci was also deeply curious about the small details that might be able to explain the human-perceived wonders he painted. 0


① A-C-B ② B-A-C ③ B-C-A ④ C-A-B ⑤ C-B-A

 


212. 20수특 T3-17

 

Many traditional sports remain important elements of contemporary national sporting cultures.


(A) In many cases, however, what are commonly assumed to be traditional sporting practices actually represent hybrid amalgamations of traditional games and imported sporting values and practices. Judo, for instance, dates from only 1882 and was developed as a modernization of traditional styles of jujitsu. 1

(B) Sumo, despite recent image problems, remains immensely popular and important in Japan. Similarly, sepak takraw and combat sports like muay thai, silat, and arnis remain permanent fixtures of Southeast Asia's sporting landscape. 0

(C) Similarly, muay thai adopted the ring, system of rounds, gloves, and weight divisions after concerns about high levels of death and injuries during the early 20th century, while sepak takraw took its current form in the 1930s, with the addition of a net and court adopted from badminton to the traditional pastime of kicking a rattan ball. 2


① A-C-B ② B-A-C ③ B-C-A ④ C-A-B ⑤ C-B-A

 


213. 20수특 T3-18

 

Although Mobile Healthcare Network applications provide numerous opportunities and benefits, they raise various security and privacy issues.


(A) Users may also worry about their critical health data being tampered with when their health data are stored in the untrusted cloud servers. 1

(B) Since the health information, eg, phenomena, health condition, emergency, is relatively sensitive for users, any inappropriate disclosure may violate user privacy and even result in property loss. 0

(C) Moreover, some malicious attackers misbehave in MHNs to disrupt the effectiveness or mislead other users' preferences. Without appropriate security and privacy protections, users may not accept MHN applications. 2


① A-C-B ② B-A-C ③ B-C-A ④ C-A-B ⑤ C-B-A

 


214. 20수특 T3-19

 

I remember as a student going to a speed-reading course of the type that was in vogue in the 1970s.


(A) The idea that you can take in the full meaning of large numbers of words in a single glance has been shown to be wrong:. yes, you can quickly understand the main point of a whole block of text at a single glance, and you can race through a book getting a pretty good idea of what the author is saying. 1

(B) We were led to believe that you could train your eye and brain to take in whole blocks of text — scores of words at a time — and that reading word by word was primitive and inefficient. It was the course that was a waste of time, however. 0

(C) But the faster you go, the more you miss. So if friends boast that they can read thick novels in an afternoon, test them on what they remember of the details. You probably can gather the plot by racing through a novel, but you'll miss much of the subtleties of the language, the scenes and the narrative. 2


① A-C-B ② B-A-C ③ B-C-A ④ C-A-B ⑤ C-B-A

 


215. 20수특 T3-20

 

Strangely enough, volcanic events have been linked to changes in the El Nifio cycle.


(A) It is not entirely clear at this point how volcanoes trigger El Ninos, but the answer may lie in the global cooling that accompanies large eruptions. According to climatologist Michael Mann, the cooling effect of volcanoes would be more notable in the western part of the Pacific Ocean, thus reducing the temperature differential between the eastern and western Pacific and setting the stage for El Nifio. 1

(B) Scientists at the National Center for Atmospheric Research have discovered a strong statistical link between El Nifio and tropical volcanic eruptions. In the years following a major eruption, they argue, the chance of El Nifio doubles compared with a standard year. 0

(C) Since the El Nifio in turn influences the formation of tropical cyclones, these findings suggest that tropical volcanoes may play a role in cyclonic weather as well. 2


① A-C-B ② B-A-C ③ B-C-A ④ C-A-B ⑤ C-B-A

 


216. 20수특 T3-21

 

On his long journey home after the Trojan War, the hero Odysseus came to an island where the goddess Circe advised him to avoid the Sirens, beautiful winged monsters whose irresistible song lured mariners to their death.


(A) The cunning hero packed his men's ears with beeswax and commanded them to tie him to the ship's mast. There he stood as they sailed into dangerous waters; the Sirens called to him, and he heard their song. 1

(B) As Circe had predicted, he longed to go to them, to cast away everything he held dear. He shouted at his men, ordered, then begged them to set him free, but the mast was strong, the rope held fast, and his men couldn't hear his pleas. And so Odysseus did not perish, but emerged on the other side of the Siren song wiser, more sensible, and prepared to complete his journey home. 2

(C) Forewarned but undaunted, Odysseus sailed into peril anyway. His plan: He would listen but not give in. 0


① A-C-B ② B-A-C ③ B-C-A ④ C-A-B ⑤ C-B-A

 


217. 20수특 T3-22

 

The concentration of large-scale economic activity has resulted in the formation of multinational companies.


(A) Such multinational companies possess considerable influence over the operations of the government of the countries in which they invest, thereby undermining the economic and political independence of such countries. In return for providing jobs and revenue derived from taxing their operations, multinational companies may demand concessions from governments as the price for their investment in that country. 1

(B) These have their headquarters in one country but their commercial activities are conducted throughout the world. Incentives for them to do this include access to raw materials and (in the case of firms locating in the third world) the availability of cheap labour. 0

(C) They may seek direct or indirect control over a country's political system to ensure that government policy is compatible with the needs of the company. If these conflict, the government may suffer: in Guatemala, for example, President Jacobo Arbenz's quarrels with the American United Fruit Company resulted in his replacement by an American-backed military government in 1954. 2


① A-C-B ② B-A-C ③ B-C-A ④ C-A-B ⑤ C-B-A

 


218. 20수특 T3-23

 

Although from very early on infants show discrimination of their mother's voice and scent, they do not exhibit a clear preference for any particular caregiver.


(A) Yet, from the beginning, babies contribute to their interactions and exchanges with others. Built-in bias to orient tow따.d, look at, and listen to certain stimuli will contribute to paying attention to and eventually developing preference for those who interact with him and provide care on a regular basis. 1

(B) Infants are not attached to their caregivers at birth. Any caregiver responding to their needs would be as effective; infants tend to respond similarly to any individual who tends to their signals or interacts with them. 0

(C) The infant uses characteristic reflexive responses in his behavioral repertoire (eg, crying, head-turning, reaching, grasping) when interacting with others. These behaviors typically have as a consequence to increase the time the baby is in proximity with those around him. 2


① A-C-B ② B-A-C ③ B-C-A ④ C-A-B ⑤ C-B-A

 


219. 20수특 T3-2425

 

Perhaps one of the most iconic memory studies was by Neisser and Harsch in 1992, in which they looked at flashbulb memories related to the Challenger explosion.


(A) Of the seven details they previously recorded, on average the students could remember only 2.95 of them. A quarter of the students scored zero out of seven, and half scored two or less. In fact, only a quarter of the students even remembered taking the survey previously. Despite their terrible recall of the event, the average score of confidence in the accuracy of their memories was as high as 4.17 out of 5. 1

(B) Other studies have also shown the lack of any relationship between confidence in a memory and its accuracy. We tend to think that vividness and confidence predict accuracy, but they don't. The clear lesson here is that we all need to be humble when it comes to the accuracy of our own memories. Failure to appreciate the true nature of memory can create great mischief. 2

(C) They gave 106 students in an introductory psychology class a questionnaire asking them to recall how they heard about the Challenger explosion, which had happened within the last twenty-four hours. They were given seven specific questions about what they were doing and how they felt at the time. Two and a half years later, the same students were given a follow-up questionnaire. In this survey they were also asked to rate their confidence in the accuracy of their memory on a scale from 1 to 5. 0


① A-C-B ② B-A-C ③ B-C-A ④ C-A-B ⑤ C-B-A

 


220. 20수특 T3-2628

 

One of the most extraordinary Olympic marathons was the one held during the 1908 London games.


(A) When John Hayes crossed the line moments later, it was he who was declared the winner. The gold medal was given to the American runner. The officials' help had disqualified Pietri. The crowd was horrified. The Olympic officials 一who should have known better 一had made a serious blunder, having been caught up in the excitement and drama of the moment. But rules are rules. Pietri may not have won the gold medal (or silver or bronze, come to that), but he was eventually presented with a special silver cup on the following day by Queen Alexandra, who was so taken by the plight and bravery of the little Italian. 2

(B) The first to cross the finishing line — in what later became known as the 'Dorando marathon' 一 was Italian pastry cook Dorando Pietri. Pietri, a small man, was an amazing runner. At 17, he once delivered an urgent message for his employer by running the 15 miles! This was the start of an impressive running career. He ran, and won, a number of marathons and then came the 1908 London Olympics. The day of the race was a hot one. 0

(C) The course of just over 26 miles ran from Windsor Castle to the White City Stadium. The streets were lined with a quarter of a million spectators. Dorando Pietri was up with the leaders from the outset and, with the finishing line in sight, he streaked into the lead, with American runner John Hayes close behind. As he entered the stadium, Pietri was suffering from heat exhaustion and fell several times inside the stadium. He kept falling and struggling back to his feet and falling down again. It was now that Olympic officials (obviously impressed by Pietri's get-up-and-go) rushed across the track, picked him up and helped him across the finishing line. 1


① A-C-B ② B-A-C ③ B-C-A ④ C-A-B ⑤ C-B-A

 


[ANSWER]
1. ② 2. ① 3. ③ 4. ⑤ 5. ⑤ 6. ⑤ 7. ④ 8. ② 9. ⑤ 10. ④


11. ④ 12. ⑤ 13. ② 14. ⑤ 15. ⑤ 16. ① 17. ④ 18. ④ 19. ② 20. ③


21. ① 22. ① 23. ④ 24. ⑤ 25. ② 26. ① 27. ② 28. ③ 29. ② 30. ③


31. ④ 32. ④ 33. ① 34. ④ 35. ① 36. ② 37. ④ 38. ② 39. ② 40. ④


41. ② 42. ② 43. ⑤ 44. ⑤ 45. ③ 46. ② 47. ① 48. ④ 49. ① 50. ①


51. ③ 52. ① 53. ② 54. ② 55. ⑤ 56. ② 57. ② 58. ③ 59. ② 60. ②


61. ② 62. ⑤ 63. ② 64. ⑤ 65. ① 66. ④ 67. ⑤ 68. ③ 69. ④ 70. ③


71. ⑤ 72. ③ 73. ④ 74. ④ 75. ⑤ 76. ⑤ 77. ④ 78. ③ 79. ④ 80. ②


81. ④ 82. ② 83. ④ 84. ② 85. ③ 86. ⑤ 87. ③ 88. ⑤ 89. ③ 90. ③


91. ④ 92. ③ 93. ① 94. ① 95. ⑤ 96. ① 97. ① 98. ④ 99. ① 100. ③


101. ① 102. ⑤ 103. ④ 104. ③ 105. ① 106. ③ 107. ③ 108. ② 109. ② 110. ①


111. ④ 112. ① 113. ⑤ 114. ② 115. ③ 116. ② 117. ① 118. ③ 119. ① 120. ③


121. ② 122. ② 123. ③ 124. ⑤ 125. ② 126. ⑤ 127. ① 128. ② 129. ③ 130. ⑤


131. ④ 132. ⑤ 133. ③ 134. ③ 135. ③ 136. ① 137. ⑤ 138. ⑤ 139. ③ 140. ①


141. ⑤ 142. ④ 143. ④ 144. ⑤ 145. ① 146. ② 147. ⑤ 148. ⑤ 149. ④ 150. ③


151. ① 152. ③ 153. ② 154. ② 155. ② 156. ④ 157. ② 158. ④ 159. ⑤ 160. ②


161. ③ 162. ① 163. ③ 164. ⑤ 165. ① 166. ③ 167. ③ 168. ① 169. ⑤ 170. ⑤


171. ⑤ 172. ④ 173. ① 174. ① 175. ② 176. ③ 177. ② 178. ① 179. ⑤ 180. ④


181. ⑤ 182. ④ 183. ⑤ 184. ① 185. ① 186. ③ 187. ⑤ 188. ⑤ 189. ④ 190. ⑤


191. ⑤ 192. ③ 193. ⑤ 194. ⑤ 195. ② 196. ① 197. ② 198. ⑤ 199. ② 200. ④


201. ② 202. ④ 203. ④ 204. ① 205. ② 206. ③ 207. ③ 208. ⑤ 209. ③ 210. ①


211. ④ 212. ② 213. ② 214. ② 215. ② 216. ④ 217. ② 218. ② 219. ④ 220. ③


 

728x90
반응형